Augmentation Mastopexy Flashcards

1
Q

A 36-year-old woman comes to the office for consultation regarding augmentation mammaplasty. She wears a size 34B brassiere and wants the size increased to a full C cup. Height is 5 ft 6 in (168 cm) and weight is 126 lb (57kg). She feels her breasts are reasonable in appearance but has been encouraged by her husband, from whom she is separated, to seek enhancement. The risks of the surgery, including loss of nipple-areola sensation and the need for prosthesis maintenance over time, are discussed. She opts to proceed with surgery, and 375-mL saline breast prostheses are placed subpectorally through inframammary fold incisions. Which of the following is most likely to cause patient dissatisfaction after the procedure?
A ) Continued separation from her husband
B ) Deflation of the breast prostheses
C ) Hypertrophy of the breast scars
D ) Inability to breast-feed
E ) Inadequate breast size

A

A ) Continued separation from her husband

How well did you know this?
1
Not at all
2
3
4
5
Perfectly
2
Q
A 47-year-old woman is referred by her primary care physician to evaluate a suspected intracapsular rupture of her prosthesis on the left identified during routine mammography. She underwent primary augmentation mammaplasty with subglandular placement of single-lumen silicone breast prostheses in 1990. Physical examination shows a smaller breast on the left. An MRI is requested. Which of the following findings on MRI is most likely to confirm the diagnosis? 
A ) Double wall sign 
B ) Linguine sign 
C ) Multiple echogenic lines 
D ) Reverse double-lumen sign 
E ) Snowstorm sign
A

B ) Linguine sign

How well did you know this?
1
Not at all
2
3
4
5
Perfectly
3
Q

Best modality for diagnosing silicone prosthesis rupture

A

MRI, mammography, ultrasonography, and CT scanning have all been used to diagnose silicone breast prosthesis rupture.Although each modality has specific strengths and weaknesses that may make a particular modality the study of choice for an individual patient, MRI of silicone breast prostheses reports the highest sensitivity and specificity for detection of silicone prosthesis rupture

How well did you know this?
1
Not at all
2
3
4
5
Perfectly
4
Q

Linguine sign

A

The linguine sign is consistent with intracapsular siliconeprosthesis rupture and represents the prosthesis shell floating in free silicone gel.

How well did you know this?
1
Not at all
2
3
4
5
Perfectly
5
Q

Double wall sign

A

The double wall sign, also known as Rigler sign, is a radiographic sign of pneumoperitoneum

How well did you know this?
1
Not at all
2
3
4
5
Perfectly
6
Q
A 26-year-old woman comes to the office for consultation regarding right mammary hypoplasia and a superiorly displaced nipple-areola complex. Examination shows a depressed right chest wall. The pectoralis major muscle is anatomically normal. Which of the following is the most likely diagnosis? 
A ) Anterior thoracic hypoplasia 
B ) Pectus carinatum 
C ) Pectus excavatum 
D ) Poland syndrome 
E ) Sternal cleft
A

A ) Anterior thoracic hypoplasia

How well did you know this?
1
Not at all
2
3
4
5
Perfectly
7
Q

Anterior thoracic hypoplasia

A

Anterior thoracic hypoplasia is a syndrome composed of an anterior chest wall depression resulting from posteriorly displaced ribs, hypoplasia of the ipsilateral breast, and a superiorly displaced nipple-areola complex. The sternum is in normal position, and the pectoralis major muscle is normal.

How well did you know this?
1
Not at all
2
3
4
5
Perfectly
8
Q

Pectus excavatum

A

Pectus excavatum is the most common congenital chest wall abnormality in which the ribs and sternum form abnormally, resulting in a concave anterior chest wall. Typically, the lower third of the sternum is involved. In the most severe form, pectus excavatum can present with the sternum adjacent to the vertebral bodies associated with cardiopulmonary abnormalities.
There is no change in development of the breast.

How well did you know this?
1
Not at all
2
3
4
5
Perfectly
9
Q

Pectus carinatum

A

n contrast, pectus carinatum is a chest wall deformity in which the sternum and ribs are forced anteriorly, creating the appearance of a pigeon chest. There is no change in development of the breast.

How well did you know this?
1
Not at all
2
3
4
5
Perfectly
10
Q

Poland syndrome

A

Poland syndrome is a congenitalanomaly characterized by a number of unilateral findings. The classic features of Poland syndrome include absence of the sternal head of the pectoralis major, hypoplasia and/or aplasia of the breast or nipple, deficiency of subcutaneous fat and axillary hair, abnormalities of the rib cage, and upper extremity anomalies. In its simplest form, Poland syndrome may present with only mild hypoplasia of the breast and lateral displacement of the nipple. Complex presentations of Poland syndrome include hypoplasiaor aplasia of the chest wall musculature (serratus, external oblique, pectoralis minor, and latissimus dorsi muscles) or total absence of the anterolateral ribs with herniation of the lung.

How well did you know this?
1
Not at all
2
3
4
5
Perfectly
11
Q

Complex presentation of Poland Syndrome

A

Complex presentations of Poland syndrome include hypoplasiaor aplasia of the chest wall musculature (serratus, external oblique, pectoralis minor, and latissimus dorsi muscles) or total absence of the anterolateral ribs with herniation of the lung

How well did you know this?
1
Not at all
2
3
4
5
Perfectly
12
Q

Sternal cleft

A

Sternal cleft is a rare congenital defect of the anterior chest wall resulting from a failure of midline fusion of the sternum. Depending on the degree of clefting, there are complete and incomplete forms. The sternal cleft is clinically significant because of the potential for the lack of protection to the heart and great vessels. Sternal clefts are not associated with aplasia or hypoplasia of the breast.

How well did you know this?
1
Not at all
2
3
4
5
Perfectly
13
Q

A 20-year-old woman comes to the office for consultation regarding augmentation mammaplasty. Height is 5 ft 4 in (163 cm) and weight is 120 lb (54 kg). Physical examination shows mammary hypoplasia. She currently wears a size 34B brassiere and would like to wear a size C brassiere. Which of the following is the most appropriate option for breast enhancement?
A ) Autologous fat transfer
B ) Breast Enhancement and Shaping System (BRAVA)
C ) Saline prostheses
D ) Smooth gel prostheses
E ) Textured gel prostheses

A

C ) Saline prostheses

How well did you know this?
1
Not at all
2
3
4
5
Perfectly
14
Q

How old must a patient be for a silicone prosthesis?

A

22 years old

How well did you know this?
1
Not at all
2
3
4
5
Perfectly
15
Q

Expcted lifespan of a saline prosthesis

A

10 years

How well did you know this?
1
Not at all
2
3
4
5
Perfectly
16
Q

BRAVA system expectations

A

The BRAVA system can increase breast size but only minimally, so it is unlikely that this would give the patient enough volume for her goal of a size C brassiere

How well did you know this?
1
Not at all
2
3
4
5
Perfectly
17
Q

A 42-year-old woman with Grade 3 ptosis of the breasts is scheduled to undergo augmentation mammaplasty and mastopexy. Which of the following operative decisions is most likely to have an adverse effect on the outcome of the procedure?
A ) Augmentation mammaplasty and use of vertical mastopexy technique
B ) Augmentation mammaplasty and use of a Wise-pattern mastopexy technique
C ) Mastopexy and placement of 450-mL saline prostheses in a dual-plane pocket
D ) Mastopexy and placement of 200-mL silicone prostheses in a subpectoral pocket
E ) Performance of the operation in two stages

A

C ) Mastopexy and placement of 450-mL saline prostheses in a dual-plane pocket

^^ large prosthesis.
A prosthesis over 250 cc is considered large

A mastopexy is designed to raise the nipple-areola complex and reshape the breast by resecting skin and tightening the parenchyma. In direct opposition to this shaping, an augmentation enlarges the volume of the breast and expands the skin envelope. Further, mastopexy techniques involve elevation of flaps thatrequire adequate vascularity, while prosthesis placement devascularizes the breast and puts direct pressure on the remaining circulation.The larger the prosthesis, the greater the adverse effect on vascularity.

How well did you know this?
1
Not at all
2
3
4
5
Perfectly
18
Q

Purpose of a mastopexy

A

A mastopexy is designed to raise the nipple-areola complex and reshape the breast by resecting skin and tightening the parenchyma.

How well did you know this?
1
Not at all
2
3
4
5
Perfectly
19
Q

The larger the prosthesis, the greater..

A

The larger the prosthesis, the greater the adverse effect on vascularity. This can lead to early problems with nipple-areola complex loss, skin flap loss, prosthesis infection and exposure, and resultant deformities.

How well did you know this?
1
Not at all
2
3
4
5
Perfectly
20
Q

Adverse considerations of larger prostheses

A

The larger the prosthesis, the greater the adverse effect on vascularity. This can lead to early problems with nipple-areola complex loss, skin flap loss, prosthesis infection and exposure, and resultant deformities.
Larger prostheses are also associated with long-term complications of soft-tissue attenuation. This results in tissue thinning, stretching, atrophy, rippling, and recurrent ptosis.

How well did you know this?
1
Not at all
2
3
4
5
Perfectly
21
Q

At what point is a prosthesis considered large

A

Despite conflicting studies, prosthesis size of 350 mL is considered the crossover to large prostheses.

How well did you know this?
1
Not at all
2
3
4
5
Perfectly
22
Q
Which of the following innervates the nipple-areola complex?
A ) Intercostal
B ) Lateral pectoral
C ) Long thoracic
D ) Supraclavicular
E ) Thoracodorsal
A

A ) Intercostal

The classic teaching ascribes nipple innervation to the fourth intercostal nerve. More recent anatomical studies have confirmed that the nipple is innervated by a rich subdermal plexus of nerves that provide both tactile and pressure sensation. This plexus receives innervation from the lateral and anterior cutaneous branches of the second to fifth intercostal nerves. This plexus explains why the nipple can retain sensation despite extensive surgical procedures.

How well did you know this?
1
Not at all
2
3
4
5
Perfectly
23
Q

The thoracodorsal innervates:

A

The thoracodorsal innervates the latissimus dorsi muscle

How well did you know this?
1
Not at all
2
3
4
5
Perfectly
24
Q

The supraclavicular innervates :

A

The supraclavicular innervates the skin of the upper breast.

How well did you know this?
1
Not at all
2
3
4
5
Perfectly
25
Q

The long thoracic innervates:

A

The long thoracic innervates the serratus anterior muscle.

How well did you know this?
1
Not at all
2
3
4
5
Perfectly
26
Q
Which of the following sequelae is more likely to result from the use of textured silicone gel prostheses rather than smooth silicone gel prostheses? 
A ) Capsular contracture 
B ) Hematoma 
C ) Malposition 
D ) Rippling
E ) Rupture
A

D ) Rippling

How well did you know this?
1
Not at all
2
3
4
5
Perfectly
27
Q

What type of prostheses are associated with more rippling?

A

The use of textured prostheses is associated with a significant rate of rippling when compared with smooth prostheses. One study reported over a two-fold increase.

Rippling is more pronounced with saline-filled prostheses.

How well did you know this?
1
Not at all
2
3
4
5
Perfectly
28
Q

Surgical approach to minimizing visible rippling

A

Visible rippling can be minimized with subpectoral implantation as well as by limiting the use of these prostheses to patients with more native breast tissue

How well did you know this?
1
Not at all
2
3
4
5
Perfectly
29
Q

Rippling occurs when?

A

Rippling occurs when the breast skin and soft tissue are thin. This rippling will worsen with time because of the skin stretching and thinning.

How well did you know this?
1
Not at all
2
3
4
5
Perfectly
30
Q

Textured prostheses vs smooth for capsular contracture.

A

Textured surface prostheses are superior to smooth prostheses in decreasing capsular contracture.

However, this advantage is minimal when using saline prostheses in a sub pectoral pocket.

How well did you know this?
1
Not at all
2
3
4
5
Perfectly
31
Q

Textured prostheses vs smooth for hematoma

A

The incidence of hematoma formation is similar for both types of prostheses.

How well did you know this?
1
Not at all
2
3
4
5
Perfectly
32
Q

Textured prostheses vs smooth for hematoma

A

Malposition rates are not higher with the use of textured prostheses.

How well did you know this?
1
Not at all
2
3
4
5
Perfectly
33
Q
A 40-year-old nulliparous woman comes to the office because she is dissatisfied with the "saggy" appearance of her breasts following a 120-lb (54-kg) weight loss. Physical examination shows bilateral Grade 3 ptosis. Which of the following additional findings on examination of the breasts is most likely in this patient? 
A ) Flatness of the upper pole 
B ) High inframammary fold 
C ) Lack of axillary fat roll 
D ) Lack of excess skin 
E ) Laterally displaced areolas
A

A ) Flatness of the upper pole

How well did you know this?
1
Not at all
2
3
4
5
Perfectly
34
Q

Breast deformities after massive weight loss

A

Patients typically present with

  • severe breast ptosis (Grade III)
  • medialization of the nipple-areola complex
  • lateralization of the breast mound
  • extension to a lateral axillary fat roll, which often extends well into the back
  • lower inframammary fold because of deflation of the entire skin and connective tissue envelope
  • more asymmetrical volume loss in the massive weight loss breast
  • more deflated and flat appearance of the breast, particularly the upper pole
  • very apparent skin laxity
How well did you know this?
1
Not at all
2
3
4
5
Perfectly
35
Q

A 24-year-old woman is undergoing endoscopic transaxillary augmentation mammaplasty. Which of the following is most appropriate to preserve sensation in the medial aspect of the upper extremity?
A ) Avoiding dissection into the axillary fat
B ) Blunt dissection near the clavicle
C ) Identification of the sensory nerves within the axilla
D ) Positioning of the prosthesis subpectorally
E ) Preservation of the lateral pectoral nerve

A

A ) Avoiding dissection into the axillary fat

Dissection within the axillary fat risks injury to these nerves with subsequent anesthesia or paresthesia of the inner arm.Identification of the nerves within the axilla requires dissection into axillary fat and risks injury to the sensory nerves.

How well did you know this?
1
Not at all
2
3
4
5
Perfectly
36
Q

During transaxillary augmentation mammaplasty, prevention of sensory changes to the medial aspect of the upper extremity:

A

During transaxillary augmentation mammaplasty, prevention of sensory changes to the medial aspect of the upper extremity requires a subdermal dissection and avoids dissection into the axillary fat.

How well did you know this?
1
Not at all
2
3
4
5
Perfectly
37
Q

Innervation to the medial upper extremity

A

Branches of the intercostobrachial and medial brachial cutaneous nerves provide sensory innervation to the medial upper extremity. Both nerves course superficially through the axillary fat posterior to the lateral border of the pectoralis major muscle.

How well did you know this?
1
Not at all
2
3
4
5
Perfectly
38
Q

Dissection within the axillary fat risks injury to sensation of:

A

Dissection within the axillary fat risks injury to these nerves with subsequent anesthesia or paresthesia of the inner arm.

How well did you know this?
1
Not at all
2
3
4
5
Perfectly
39
Q

A 35-year-old woman comes to the office for consultation regarding augmentation mammaplasty. A preoperative mammogram is most indicated if the patient’s history includes which of the following?
A ) A grandmother diagnosed with breast cancer at age 73 years
B ) A mother diagnosed with breast cancer at age 45 years
C ) Personal history of breast cysts
D ) Personal history of fibroadenoma
E ) A sister diagnosed with ovarian cancer

A

B ) A mother diagnosed with breast cancer at age 45 years

How well did you know this?
1
Not at all
2
3
4
5
Perfectly
40
Q

Familial breast cancer vs hereditary breast cancer

A

Familial breast cancer most likely results from changes in multiple low penetrance genes coupled with environmental influences.
Hereditary breast cancer results in high penetrance mutation in a single gene.

How well did you know this?
1
Not at all
2
3
4
5
Perfectly
41
Q

Familial breast cancer

A

Familial breast cancer most likely results from changes in multiple low penetrance genes coupled with environmental influences.

Familial breast cancer is relatively common and conveys a modest elevation in risk compared with genetic breast cancer, which is rare but associated with high risk.

How well did you know this?
1
Not at all
2
3
4
5
Perfectly
42
Q

Hereditary breast cancer

A

Hereditary breast cancer results in high penetrance mutation in a single gene.

How well did you know this?
1
Not at all
2
3
4
5
Perfectly
43
Q

Risk of individual vs general population, whose first degree relative has had bilateral breast cancer

A

Individuals whose first-degree relatives have bilateral breast cancer have an increased risk of 5.5 times the normal population.

How well did you know this?
1
Not at all
2
3
4
5
Perfectly
44
Q
A 48-year-old woman comes to the office because she is dissatisfiedwith the "sagging" appearance of her breasts. Physical examination shows the location of the nipples 1 cm above the inframammary fold bilaterally. The majority of breast tissue is below the fold. Which of the following is the most likely diagnosis?
A ) Grade 1 ptosis
B ) Grade 2 ptosis
C ) Grade 3 ptosis
D ) Pseudoptosis
A

D ) Pseudoptosis

How well did you know this?
1
Not at all
2
3
4
5
Perfectly
45
Q

Who defined the classification of ptosis?

A

Regnault defined the degree of ptosis by evaluating the relationship of the nipple to the inframammary fold

How well did you know this?
1
Not at all
2
3
4
5
Perfectly
46
Q

Pseudoptosis

A

In pseudoptosis, the nipple is above or at the level of the inframammary fold, with the majority of the breast tissue below. This gives the impression of ptosis.

How well did you know this?
1
Not at all
2
3
4
5
Perfectly
47
Q

Grade 1 ptosis

A

In Grade 1, or mild ptosis, the nipple is within 1 cm of the level of the inframammary fold and above the lower contour of the breast and skin envelopes.

How well did you know this?
1
Not at all
2
3
4
5
Perfectly
48
Q

Grade 2 ptosis

A

In Grade 2, or moderate ptosis, the nipple is 1 to 3 cm below the inframammary fold but above the lower contour of the breast and skin envelopes.

How well did you know this?
1
Not at all
2
3
4
5
Perfectly
49
Q

Grade 3 ptosis

A

In Grade 3, or severe ptosis, the nipple is more than 3 cm below the inframammary fold and below the lower contour of the breast and skin envelopes.

How well did you know this?
1
Not at all
2
3
4
5
Perfectly
50
Q
A 25-year-old woman comes to the office because she has a lump in her right armpit. She reports that the lump increases in size and becomes tender during her period. She also says that it restricts arm movement and interferes with her ability to play tennis, especially during menses. Examination shows a 4 * 4-cm, soft, mobile mass in the right axilla that is tender to palpation. There is no evidence of firmness or palpable nodules within the mass. Which of the following is the most appropriate next step in management?
A ) Excision of axillary tissue
B ) Fine-needle aspiration
C ) Incisional biopsy
D ) Mammogram
E ) Sentinel lymph node biopsy
A

A ) Excision of axillary tissue

How well did you know this?
1
Not at all
2
3
4
5
Perfectly
51
Q

Management of axillary breast tissue

A

Axillary accessory breast tissue should be removed surgically.

How well did you know this?
1
Not at all
2
3
4
5
Perfectly
52
Q

Axillary breast tissue

A

Found in 0.4% to 6% of women and may be asymptomatic, cause pain, restrict arm movement, or cause cosmetic problems or anxiety. There have been reports of malignant degeneration

How well did you know this?
1
Not at all
2
3
4
5
Perfectly
53
Q

Why should axillary breast tissue be removed?

A

There have been reports of malignant degeneration of this accessory breast tissue, and the current recommendations are for simple excision through an axillary incision.

How well did you know this?
1
Not at all
2
3
4
5
Perfectly
54
Q
A 24-year-old woman comes to the office one year after undergoing secondary augmentation mammaplasty because she reports that with manipulation she “can move each breast to the other side.” Physical examination shows that each breast prosthesis can be moved across the chest midline. Which of the following is the LEAST likely cause of this finding?
A ) Multiple procedures
B ) Preexisting chest wall deformity
C ) Prostheses with large base diameter
D ) Saline prostheses
E ) Subpectoral positioning
A

D ) Saline prostheses

How well did you know this?
1
Not at all
2
3
4
5
Perfectly
55
Q

Symmastia

A

Synmastia is defined as any situation in which the breast prosthesis crosses the midline, even if it is only on one side.

How well did you know this?
1
Not at all
2
3
4
5
Perfectly
56
Q

Assotiations vs symmastia

A

There is no correlation with the use of either silicone-or saline-filled prostheses. The complication is more common in cases in which large prostheses with large base diameters are used, in multiple successive enlargement procedures, when there is a preexisting chest wall deformity, and with the subpectoral positioning of prostheses.

How well did you know this?
1
Not at all
2
3
4
5
Perfectly
57
Q

A 35-year-old woman, gravida 3, para 3, comes to the office for consultation about augmentation mammaplasty with gel prostheses. During the visit, she inquires about the safety of breast-feeding after augmentation mammaplasty with silicone prostheses. She should be informed that the silicone levels in her breast milk after the augmentation will be which of the following?
A ) Similar to the levels in the milk of patients with no prostheses
B ) Higher than the levels in the milk of patients with no prostheses
C ) Similar to the levels in commercially available infant formula
D ) Higher than the levels in commercially available infant formula

A

A ) Similar to the levels in the milk of patients with no prostheses

How well did you know this?
1
Not at all
2
3
4
5
Perfectly
58
Q

Silicone levels in breast milk after augmentation

A

Not significantly different from women without augmentation

How well did you know this?
1
Not at all
2
3
4
5
Perfectly
59
Q

Silicone levels in infant formulas

A

Mean silicone levels in breast milk of augmented and nonaugmented women were not significantly different (55 ng/mL and 51 ng/mL, respectively). The silicon particle was used as a proxy of silicone. Interestingly, silicone levels were significantly higher in commercially available infant formulas (4402.5 ng/mL).

How well did you know this?
1
Not at all
2
3
4
5
Perfectly
60
Q

Which of the following most appropriately describes the biomechanical characteristic specifically designed to minimize gel diffusion in a third-generation silicone prosthesis?
A ) Increased cross-linking of silicone elastomer
B ) Increased molecular weight of silicone gel
C ) Internal barrier coating
D ) Texturing of prosthesis surface

A

C ) Internal barrier coating

How well did you know this?
1
Not at all
2
3
4
5
Perfectly
61
Q

Modification in 3rd gen silicone breast prostheses to reduce silicone bleed

A

While a number of biomechanical properties were altered, the modification that was specifically designed to reduce silicone bleeding was the addition of an inner barrier on the elastomer shell. This barrier changed the solubility characteristics of the shell, thus inhibiting the diffusion of silicone through it.

How well did you know this?
1
Not at all
2
3
4
5
Perfectly
62
Q

Modification in 3rd gen silicone breast prostheses to reduce silicone bleed: barrier difference between companies

A

McGhan Medical released a prosthesis that had a diphenyl silicone copolymer barrier layer between an inner and outer layer of high-performance elastomer (Intrashiel).
The Dow-Corning Silastic II prosthesis had a fluorosilicone copolymer layer to restrict silicone bleed

How well did you know this?
1
Not at all
2
3
4
5
Perfectly
63
Q

First generation breast silicone prostheses

A

First-generation prostheses had thick shell walls, viscous gel, and Dacron patches. Hardness and contracture were major complaints.

How well did you know this?
1
Not at all
2
3
4
5
Perfectly
64
Q

Second generation breast silicone prostheses

A

To address the hardness/contracture of 1st generation silicone prostheses, second-generation prostheses were developed (1973-1985), which had thinner walls and lower viscosity gel. The result was a softer, more natural-feeling prosthesis in the early postoperative period. Second-generation prostheses had the highest rates of rupture, bleed, and capsular contracture

How well did you know this?
1
Not at all
2
3
4
5
Perfectly
65
Q

3rd generation breast silicone prostheses

A

To address the rupture, bleed, and capsular contracture of 2nd gen prostheses, third-generation prostheses reintroduced thicker shells and more cohesive gel.

How well did you know this?
1
Not at all
2
3
4
5
Perfectly
66
Q

Increasing the cross-linking of the silicone elastomer:

A

Increasing the cross-linking of the silicone elastomer strengthens and thickens the wall of the prosthesis

How well did you know this?
1
Not at all
2
3
4
5
Perfectly
67
Q
A 26-year-old woman who underwent augmentation mammaplasty six months ago comes to the office because she has numbness of the right nipple. The most likely cause is injury to which of the following intercostal nerves?
A ) Second
B ) Third
C ) Fourth
D ) Fifth
E ) Sixth
A

C ) Fourth

How well did you know this?
1
Not at all
2
3
4
5
Perfectly
68
Q

Innervation of the nipple

A

The lateral cutaneous branches of the third through fifth intercostal nerves and the anterior cutaneous branches of the second through fifth intercostal nerves all contribute to nipple supply.

How well did you know this?
1
Not at all
2
3
4
5
Perfectly
69
Q

Course of the 4th lateral cutaneous branch to the nipple

A

The lateral cutaneous branch of the fourth intercostal nerve has been traced into the nipple and found to have two branches. The deep branch passes inferolaterally on the pectoralis major fascia before coursing up into the areola, whereas the superficial branch passes up through the superficial parenchyma

How well did you know this?
1
Not at all
2
3
4
5
Perfectly
70
Q
Which of the following is the most likely site of ectopicbreast tissue in a patient with ectopic polymastia?
A ) Axilla
B ) Costal margin
C ) Dorsal thigh
D ) Pubis
E ) Vulva
A

C ) Dorsal thigh

How well did you know this?
1
Not at all
2
3
4
5
Perfectly
71
Q

Ectopic breast tissue

A

Ectopic breast tissue is found outside the milk line at such sites as the scalp, ear, back, shoulder, epigastrium, and posterior or dorsal thigh.

How well did you know this?
1
Not at all
2
3
4
5
Perfectly
72
Q

Accessory polymastia

A

Accessory polymastia occurs along the milk line. Greater than 90% of accessory breast tissue is localized to the chest region. The axilla, groin, vulva, and medial thigh may also be affected as well as regions above or below the normal breast, such as the costal margin.

How well did you know this?
1
Not at all
2
3
4
5
Perfectly
73
Q

Ectopic vs accessory polymastia

A

Ectopic: outside the milk line
Accessory: along the milk line

How well did you know this?
1
Not at all
2
3
4
5
Perfectly
74
Q
A 15-year-old girl is brought to the office for consultation regarding correction of breast asymmetry. Physical examination shows asymmetry of breast size and shape. The right breast is 90% smaller than the left breast. The right pectoralis major muscle is normal. The right nipple is present but smaller than the left nipple. Which of the following is the most likely diagnosis?
A ) Amastia
B ) Amazia
C ) Athelia
D ) Jeune syndrome
E ) Poland syndrome
A

B ) Amazia

How well did you know this?
1
Not at all
2
3
4
5
Perfectly
75
Q

Amazia

A

Absence of the breast glandular tissue only.

Amazia can result from surgical removal of the breast bud, radiation, or congenital absence. I

How well did you know this?
1
Not at all
2
3
4
5
Perfectly
76
Q

Athelia

A

absence of the nipple alone

How well did you know this?
1
Not at all
2
3
4
5
Perfectly
77
Q

Amastia

A

Amastia is the absence of both breast and nipple.

How well did you know this?
1
Not at all
2
3
4
5
Perfectly
78
Q
A 36-year-old woman comes to the office for consultation regarding “sagging” breasts 10 years after undergoing bilateral augmentation mammaplasty with subglandular placement of saline prostheses. Physical examination shows Grade 2 ptosis and an axillary scar. A mastopexy with capsulotomy and replacement of prostheses is planned. Which of the following pedicles is LEAST likely to preserve the blood supply to the nipple-areola complex?
A ) Inferior
B ) Medial
C ) Superior
D ) Superolateral
E ) Superomedial
A

A ) Inferior

How well did you know this?
1
Not at all
2
3
4
5
Perfectly
79
Q

Where should pedicles be based for secondary augmentation mastopexy

A

Gravity causes most soft-tissue thinning and atrophy to eventually occur in the inferior pole of the augmented breast. Therefore, in secondary mastopexy augmentation procedures, blood supply to the nipple-areola complex should generally rely on a medial, superior, superomedial, or superolateral pedicle

How well did you know this?
1
Not at all
2
3
4
5
Perfectly
80
Q

A 28-year-old woman is scheduled to undergo release of severe cicatricial contraction six months after removal of an infected breast prosthesis. Preoperative physical examination shows that soft tissue is required in the inframammary area. Closure with a submammary flap is planned. Which of the following vessels is most likely to supply blood to this flap?
(A)Internal mammary perforators
(B)Lateral thoracic artery
(C)Superficial inferior epigastric artery
(D)Thoracoacromial perforators
(E)Thoracodorsal perforators

A

(A)Internal mammary perforators

How well did you know this?
1
Not at all
2
3
4
5
Perfectly
81
Q

Contraction after breast augmentation infection

A

The sequelae of infection in breast augmentation can be severe cicatricial contraction of the inferior pole of the breast.

How well did you know this?
1
Not at all
2
3
4
5
Perfectly
82
Q

Timeline for operative intervention after infection of breast augmentation

A

eoperation can be considered after an appropriate interval of six months, which allows for resolution of inflammation and scar maturation.

How well did you know this?
1
Not at all
2
3
4
5
Perfectly
83
Q

Source for additional soft tissue, if required, for operative management of infected breast augmentation

A

If additional soft tissue is required, submammary flaps (with good color and texture match) can be used from the medial or lateral base of the breast. The blood supply is based medially on perforators of the internal mammary or superior epigastric arteries and laterally from perforators of the intercostal vessels.

How well did you know this?
1
Not at all
2
3
4
5
Perfectly
84
Q

Blood supply for submammary flaps

A

The blood supply is based medially on perforators of the internal mammary or superior epigastric arteries and laterally from perforators of the intercostal vessels.

How well did you know this?
1
Not at all
2
3
4
5
Perfectly
85
Q

Blood supply for lateral chest flap

A

The lateral thoracic artery is the blood supply for a lateral chest flap

How well did you know this?
1
Not at all
2
3
4
5
Perfectly
86
Q

A 36-year-old woman comes to the office for consultation regarding mastopexy. She will not consider use of prostheses and is concerned about the length of the scars. Photographs of the breasts are shown. (Grade 2 ptosis)
Which ofthe following types of mastopexy is most appropriate for this patient?
(A)Circumareolar
(B)Crescent
(C)Vertical
(D)Wise-pattern

A

(C)Vertical

The most appropriate management for the patient described, who has grade 2 ptosis of the breasts, is a vertical mastopexy. The procedure will leave periareolar and vertical scars but will give the patient a longer-lasting result than a periareolar procedure

How well did you know this?
1
Not at all
2
3
4
5
Perfectly
87
Q

Treatment for grade 1 ptosis (without augmentation)

A

If the patient does not desire implants, a dermal or crescent mastopexy, which involves excision of a crescent-shaped area of skin above the areola, may be necessary. Circumareolar mastopexy, which involves concentric excision of skin and leaves no vertical scar beneath the areola, is also adequate for grade 1 ptosis.

Periareolar resections without implant placement tend to flatten the shape of the breast.

How well did you know this?
1
Not at all
2
3
4
5
Perfectly
88
Q

Treatment for grade 2 ptosis

A

A vertical or infraareolar mastopexy is ideal for grade 2 or moderate ptosis.

How well did you know this?
1
Not at all
2
3
4
5
Perfectly
89
Q

Treatment for grade 3 ptosis

A

Wise-pattern mastopexy is appropriate for grade 3 ptosis with large amounts of skin excess, but the procedure will leave an inverted T-shaped scar.

How well did you know this?
1
Not at all
2
3
4
5
Perfectly
90
Q
A 26-year-old woman is undergoing subglandular implantation of saline breast prostheses. Pinch test of the superior pole shows a thickness of 1 cm. This patient is most at risk for which of the following complications?
(A)Capsular contracture
(B)Double-bubble deformity
(C)Infection
(D)Numbness
(E)Wrinkling
A

(E)Wrinkling

Infection and numbness are possible complications of implant surgery but are less common than wrinkling in a thin patient with subglandular implants.

How well did you know this?
1
Not at all
2
3
4
5
Perfectly
91
Q

Sub glandular breast prosthesis placement

A

The subglandular placement of breast prostheses has both advantages and disadvantages. Because the prostheses are closer to the skin, the patient’s native skin and subcutaneous fat layer are the only coverage and must be carefully evaluated. Subglandular implants are less painful than other methods, and they age well with the breast.

How well did you know this?
1
Not at all
2
3
4
5
Perfectly
92
Q

Pinch test for subglandular implant placement

A

When evaluating the superior pole of the breast for adequate soft-tissue coverage, a minimum pinch test of 2 cm is recommended. Soft-tissue thickness of less than 2 cm will increase the chance of rippling and wrinkling with a subglandular placement. If the pinch test is less than 2 cm, submuscular placement is recommended for greater soft-tissue coverage of the prosthesis

How well did you know this?
1
Not at all
2
3
4
5
Perfectly
93
Q

Double-bubble deformity

A

A double-bubble deformity occurs when the native glandular tissue lies at the lower pole of an implant, or when an implant falls below the inframammary fold.

How well did you know this?
1
Not at all
2
3
4
5
Perfectly
94
Q

A 26-year-old woman comes to the office because she has pain and tenderness of the right breast three weeks after undergoing augmentation mammaplasty. The patient is satisfied with the appearance of the prostheses and does not want them permanently removed. Temperature is 39.0°C (102.2°F). She has chills and sweating. Physical examination shows induration of the right breast and drainage from the surgical incision. Gram stain of the drainage shows gram-negative rods. Which of the following is the most appropriate management?
(A)Immediate hospitalization for intravenous antibiotic therapy
(B)Oral antibiotic therapy and follow-up evaluation in three days
(C)Removal of the implant, irrigation of the pocket, capsule debridement, and immediate reinsertion of new implant
(D)Removal of the implant, irrigation of the pocket, capsule debridement, and reinsertion of new implant in six months
(E)Removal of the implant, irrigation of the pocket, and immediate reinsertion of new implant

A

(D)Removal of the implant, irrigation of the pocket, capsule debridement, and reinsertion of new implant in six months

The patient described has a severe infection with an elevated temperature, chills, diaphoresis, and signs of cellulitis. The Gram stain of the leaking fluid implicates involvement of the implant pocket. An infection of the implant pocket is difficult to control without removal of the implant. The most appropriate management is removal of the implant, irrigation of the pocket, debridement of the capsule, and reinsertion of an implant several months later. This approach minimizes the costs and risks associated with prolonged salvage attempts

Administration of antibiotics, either oral or intravenous, without drainage of the infected pocket is not likely to eradicate the infection. This treatment approach is indicated only for a superficial infection without involvement of the periprosthetic space

How well did you know this?
1
Not at all
2
3
4
5
Perfectly
95
Q

Management of infected implant

A

Patients with a severe implant infection may present with an elevated temperature, chills, diaphoresis, and signs of cellulitis. A Gram stain of the leaking fluid implicates involvement of the implant pocket. An infection of the implant pocket is difficult to control without removal of the implant. The most appropriate management is removal of the implant, irrigation of the pocket, debridement of the capsule, and reinsertion of an implant several months later. This approach minimizes the costs and risks associated with prolonged salvage attempts.

How well did you know this?
1
Not at all
2
3
4
5
Perfectly
96
Q

Management of superficial infection over a breast prosthesis

A

Administration of antibiotics, either oral or intravenous, without drainage of the infected pocket is not likely to eradicate the infection. This treatment approach is indicated only for a superficial infection without involvement of the periprosthetic space

97
Q

Removal of an infected implant: management of capsule

A

Removal of the implant, irrigation of the pocket, capsule debridement, and immediate reinsertion of new implants can be performed. However, it should be noted that this approach can increase costs and risks, such as capsular contracture. In any case of implant removal, debridement of as much of the capsule as possible is recommended.

98
Q

A 22-year-old woman comes to the office for consultation regarding correction of breast asymmetry. She says the problem is with the left breast; she is happy with the size and shape of the right breast. Physical examination shows narrowing of the left breast at the base. At the mid portion, the inframammary fold of the left breast is higher than that of the right breast. The left areola is enlarged. The cup size of the left breast is B, and the cup size of the right breast is C. Which of the following is the most appropriate management?
(A)Augmentation mammaplasty of the left breast with radial scoring and areola reduction
(B)Mastopexy of the left breast using a Wise-pattern technique with lowering of the inframammary fold
(C)A pedicled TRAM flap to the left breast
(D)Vertical reduction mammaplasty of the right breast

A

(A)Augmentation mammaplasty of the left breast with radial scoring and areola reduction

The patient described has a tubular breast deformity. The most appropriate management is augmentation mammaplasty of the left breast with radial scoring and areola reduction.

99
Q

Management of tubular breast deformity

A

The most appropriate management is augmentation mammaplasty of the breast with radial scoring and areola reduction.

100
Q

A 35-year-old woman, gravida 4, para 4, has undergone uncomplicated augmentation mammaplasty with silicone gel–filled prostheses. According to prosthesis manufacturers and the United States Food and Drug Administration, which of the following is the recommended schedule of MRI screening to detect prosthesis rupture?
(A)One year after surgery, then every two years
(B)Two years after surgery, then every two years
(C)Two years after surgery, then every three years
(D)Three years after surgery, then every two years
(E)Three years after surgery, then every three years

A

(D)Three years after surgery, then every two years

101
Q

FDA screening recommendations for screening for silent silicone implant rupture, updated in 2011

A

FDA recommends that women with silicone gel-filled breast implants get screenings for silent ruptures three years after they get implants and every two years after that.

102
Q

Symptoms of silent silicone implant rupture

A

Symptoms of silent rupture include hard knots or lumps surrounding the prosthesis or in the armpit; a change or loss of size or change in shape of the breast or prosthesis; and pain, tingling, swelling, numbness, burning,or hardening of the breast.

103
Q

A 35-year-old woman comes to the office because she is unhappy with the appearance of her right breast. She underwent implantation of a silicone prosthesis in the right breast 15 years ago to correct asymmetry. She has required three revision surgeries for severe capsular contracture. She wants to have the implant removed, but she also wants to retain symmetry of the breasts. Examination shows grade 3 capsular contracture in the right breast. Which of the following is the most appropriate management of the right breast after removal of the implant?
(A) Excision of the capsule alone
(B)Excision of the capsule and injection of aspirated fat
(C)Excision of the capsule and insertion of a saline implant
(D)Excision of the capsule and reconstruction with an autologous flap

A

(D)Excision of the capsule and reconstruction with an autologous flap

The patient described requires implant removal and capsulectomy to remove the firm scar tissue surrounding the implant. Reconstruction with an autologous flap is the most appropriate management to maintain volume and avoid capsular contracture.

104
Q

A 32-year-old woman who underwent submuscular placement of smooth 240-ml saline breast prostheses 10 years ago comes to the office for consultation regarding replacement of the implants. She says she wants her breasts to be two cup sizes larger. Currently, she wears a size 36C brassiere. Physical examination shows good aesthetic outcome and no evidence of capsular contracture or rippling. Which of the following is the most likely adverse effect of implant exchange in this patient?
(A)Inability to breast-feed
(B)Increased capsular contracture
(C)Increased risk of breast cancer
(D)Increased risk of collagen vascular disease
(E)Shrinkage of breast tissue

A

(E)Shrinkage of breast tissue

Shrinkage of breast tissue occurs with all prostheses; the larger the prosthesis, the more shrinkage that occurs.

105
Q

Negative effects following reoperation for dissatisfaction with breast size after initial implantation of breast prostheses

A

Reoperation for dissatisfaction with breast size after initial implantation of breast prostheses must be informed of the following long-term negative effects: thinning of tissue, stretching of tissue, shrinkage of breast tissue, additional and more rapid sagging, palpable implant edges and shell, visible implant edges, visible traction rippling, and possible additional sensory loss.

106
Q
A 52-year-old woman comes to the office because she has had progressive hardening of the left breast for the past two months. She underwent augmentation mammaplasty with implantation of saline-filled prostheses three years ago. On physical examination, the left breast is firm and elevated compared with the right. It is cool and painful. The patient’s symptoms are most consistent with which of the following Baker classification levels?
(A) I
(B) II
(C) III
(D) IV
A

(D) IV

107
Q

Capsular contracture rating scale

A

Baker Classification of Capsular Contracture After Breast Augmentation:

Class I: Normal breast; augmentation is not noticeable

Class II: Minimal contracture; the implant can be palpated but is not visible

Class III: Moderate contracture; the implant is palpable and distortion is visible

Class IV: Severe contracture; the breast is distorted, hard, cool, and painful

108
Q

In patients who have undergone breast augmentation, capsular contracture occurs in approximately:

A

In patients who have undergone breast augmentation, capsular contracture occurs in approximately 5% to 7% at one year after the procedure and in approximately 18% at three years postoperatively

109
Q

Common reasons for augmentation revision:

A

Common reasons for augmentation revision include patient request for size or shape change (30%), leakage or deflation (20%), contracture (18%), wrinkling (5%), and infection (5%).

110
Q

A 38-year-old woman comes to the office for consultation regarding surgical correction of sagging of the breasts. She breast-fed three children during the past five years; her youngest child was weaned two years ago. Physical examination shows second-degree ptosis. For this patient, which of the following is an advantage of mastopexy with augmentation over mastopexy alone?
(A) Decreased risk of loss of nipple sensation
(B) Decreased risk of nipple malposition over time
(C) Decreased stretch deformity of surgical scars
(D) Increased longevity of correction of ptosis
(E) Increased upper pole volume

A

(E) Increased upper pole volume

The combination of implantation of a prosthesis with mastopexy can enhance the size and contour of the breast. This procedure often reduces the length of the incisions required to correct the ptosis because of the volume enhancement delivered by the implant.

111
Q

Correction of ptosis with prosthesis and mastopexy - effect of adding prosthesis on scar

A

The combination of implantation of a prosthesis with mastopexy can enhance the size and contour of the breast. This procedure often reduces the length of the incisions required to correct the ptosis because of the volume enhancement delivered by the implant.

112
Q

Correction of ptosis with prosthesis and mastopexy - effect of adding prosthesis on complication

A

Degree of loss of sensation: no change

Recurrence of ptosis: The weight of the prosthesis places additional tension at the site of incision, causing more rapid recurrence of ptosis. This is especially true for larger prostheses placed in the sub glandular position

Risk of nipple malposition: There is an increased risk of nipple malposition because the nipple is moved at the same time as the implant

113
Q
A 34-year-old woman comes to the office for consultation regarding breast augmentation. She is 5 ft 2 in tall and wears a size 34A brassiere. Submuscular implantation of 300-ml prostheses is planned. She asks for information about silicone versus saline implants. The primary advantage of using saline-filled implants is which of the following?
(A) Easier detection of rupture
(B) Increased softness
(C) Less capsular contracture
(D) Less leakage
(E) Less wrinkling
A

(A) Easier detection of rupture

Although both silicone and saline implants will rupture eventually, a saline rupture is more easily detected because the saline is resorbed into the body. The breast will be smaller in volume with prominent wrinkling. A ruptured silicone implant retains its volume and is more difficult to detect. Subtle changes, such as decreased upper pole fullness or increased softness, may be the only clues to silicone implant rupture on physical examination. Ultrasonography or MRI may be needed to make the diagnosis.

114
Q

Ease of placement: Saline vs silcone

A

Saline implants are easier to place because they can be inflated after placement and are placed through narrow long tunnels if warranted.

115
Q

Contracture, wrinkling, leakage rates of saline vs silicone prostheses

A

Both types produce similar rates

116
Q
A 55-year-old woman has nipples located 8 cm inferior to the inframammary fold and at the lowest point of the breast contour. Which of the following best describes the degree of breast ptosis in this patient?
(A) Glandular ptosis
(B) Grade 1
(C) Grade 2
(D) Grade 3
(E) Pseudoptosis
A

(D) Grade 3

117
Q
A 30-year-old woman undergoes augmentation mammaplasty with silicone gel prostheses. During the procedure, smooth prostheses are positioned subglandularly. The subglandular placement increases this patient's risk of which of the following complications?
(A) Capsular contracture
(B) Double-bubble appearance
(C) Infection of the implant
(D) Rippling of the implant
(E) Rupture of the implant
A

(A) Capsular contracture

118
Q

Surgical placement to decrease implant contracture

A

Submuscular placement is a well-established method of reducing the rate of contracture

119
Q

Development of capsular contracture after sub glandular placement

A

Development of capsular contracture is clearly more common in the first two years after subglandular implantation, regardless of the implant type.

120
Q

Difference in capsular contracture after sub pectoral placement

A

The large difference in the rate of contracture between textured and smooth prostheses in the subglandular position seems to be negligible in subpectoral placement; both types of implant have low contracture rates

121
Q

A 24-year-old woman comes to the office for consultation regarding surgical correction of the breast deformity shown above (tuberous breast deformity on the left). Which of the following is the most appropriate management?
(A) Augmentation with Wise-pattern mastopexy of both breasts
(B) Augmentation with periareolar mastopexy of both breasts
(C) Latissimus dorsi myocutaneous flap reconstruction of the left breast and periareolar mastopexy of the right breast
(D) Extended dorsi myocutaneous flap reconstruction of the left breast and periareolar reduction of the right breast
(E) Transaxillary augmentation of the left breast and periareolar mastopexy of the right breast

A

(B) Augmentation with periareolar mastopexy of both breasts

122
Q

Tuberous breast deformity

A

Herniation of the breast tissue into the nipple-areola complex with a cylindrical projection accompanied by a relatively large areola; deficiency of the lower pole of the breast in both vertical and horizontal axes; and hypoplasia. P

123
Q

Periaolar approach and tuberous breast deformity vs transaxillary augmentation

A

Periareolar mastopexy with augmentation will give access for radial-releasing incisions, which will allow expansion of the base of the breast and simultaneous areolar reduction.

Transaxillary augmentation of the breast will not correct the nipple-areola complex or the constricted base.

124
Q

In implantation of saline breast prostheses, which of the following fill levels is most likely to result in rupture due to fold flaw?
(A) Above the manufacturer’s recommended maximum
(B) Below the manufacturer’s recommended minimum
(C) Between the manufacturer’s recommended minimum and maximum
(D) Manufacturer’s recommended maximum
(E) Manufacturer’s recommended minimum

A

(B) Below the manufacturer’s recommended minimum

125
Q

Decreasing fold-flaw failures vs saline implant longevity

A

Based on engineering principles, studies have shown that implant longevity requires an adequate fill level to decrease fold-flaw failures and premature failures that result from underfilling (filling at or below the manufacturer’s recommended minimum level). They have also shown that filling the implants to their least-wrinkled fill level increases implant longevity and decreases premature failure. This generally requires overfilling exceedingthe manufacturer’s recommended maximum level.
However, this technique, known as ―overfill,‖ is not recommended

126
Q

At lower fill levels for saline implant breast augmentation

A

At lower fill levels, implants are softer and more sloping in contour but tend to wrinkle more and have a demonstrably shorter life span because of stress caused by wrinkling. As implant volume increases, palpable and visible wrinkling decreases and longevity increases.

127
Q
A healthy 24-year-old woman undergoes bilateral cosmetic breast augmentation with subglandular saline implants. Which of the following percentages best represents this patient’s 10-year risk for reoperation because of an implant-related indication?
(A) 5%
(B) 25%
(C) 50%
(D) 75%
(E) 95%
A

(B) 25%

128
Q

After breast augmentation with saline implants, the 10-year risk for reoperation for any implant-related indication:

A

After breast augmentation with saline implants, the 10-year risk for reoperation for any implant-related indication is about 25%. Implant-related indications include deflation of the implant, capsular contracture, hematoma, wound infection, and seroma.

129
Q

A woman comes to the office for consultation regarding explantation of breast prostheses without replacement. In this patient, which of the following quantifications best determines whether mastopexy will be needed in addition to removal of the prostheses?
(A) Amount of breast tissue overlying the prostheses
(B) Degree of preoperative ptosis
(C) Position of the prostheses
(D) Size of the areolae
(E) Type of implants

A

(B) Degree of preoperative ptosis

130
Q

Most important factor in determining whether a patient will need mastopexy after explantation of breast protheses

A

The degree of preoperative breast ptosis is the most important factor in determining whether a patient will need mastopexy after explantation of breast protheses. Because breast ptosis remains relatively unchanged or worsens postoperatively, patients with grade II or III ptosis are excellent candidates for breast contouring procedures

131
Q

What determines the safety of a breast contouring procedure done simultaneously with explantation

A

The amount of breast tissue overlying the prostheses determines the safety of a breast contouring procedure done simultaneously with explantation. In general, at least 4 cm of breast tissue should be present to allow for adequate vascularity of the skin and separated glandular-nipple flap used for breast contouring, as assessed by the superior and inferior ―breast pinch‖ test

132
Q

A 32-year-old woman is undergoing breast augmentation. Which of the following antibiotic solutions is most appropriate for irrigation of the breast pocket?
(A) Bacitracin, cefazolin, gentamicin
(B) Polymyxin B, gentamicin, cefazolin
(C) 10% Povidone-iodine, gentamicin, cefazolin
(D) 50% Povidone-iodine

A

(A) Bacitracin, cefazolin, gentamicin

Povidone-iodine (50%) does not provide optimal broad-spectrum activity, and contact of the implant with povidone-iodine is contraindicated. The triple combination of 10% povidone-iodine, gentamicin, and cefazolin is a viable alternative. However, if thiscombination is to be used, pockets would need to be irrigated clear after its instillation; therefore, this is not the optimal choice.

133
Q

Breast pocket irrigation has been advocated for breast augmentation for:

A

Breast pocket irrigation has been advocated for many years to decrease the incidence of capsular contracture and periprosthetic breast implant infection.

134
Q

Irrigation solution for breast implant procedures

A

Multiple organisms have been cultured around breast implants, and in vitro studies have demonstrated that a combination triple antibiotic (10% povidone-iodine, gentamicin, cefazolin) combination provided improved broad-spectrum activity against the bacteria commonly cultured around breast implants compared to other combinations.

135
Q
Compared with traditional (nonendoscopic) transaxillary submuscular techniques for breast augmentation, endoscopic techniques are associated with a decreased risk for which of the following? 
(A) Capsular contracture 
(B) Deflation
(C) Hypertrophic scarring
(D) Infection
(E) Malpositioning of the implant
A

(E) Malpositioning of the implant

136
Q

Traditional transaxillary breast augmentation - advantages and disadvantages

A

Transaxillary breast augmentation is an established technique that allows the surgeon to make the incision in an aesthetically acceptable area, where it can be hidden. However, one disadvantage of traditional transaxillary augmentation is a lack of visualization of the implant pocket, necessitating blind, blunt dissection at the origin of the pectoral muscle. This limitation may result in improper implant placement, leading to malpositioning of the implant and poor aesthetic results in some patients.

137
Q

Advantage of adding endoscopic technique to traditional transaxillary augmentation

A

One disadvantage of traditional transaxillary augmentation is a lack of visualizationof the implant pocket, necessitating blind, blunt dissection at the origin of the pectoral muscle. This limitation may result in improper implant placement, leading to malpositioning of the implant and poor aesthetic results in some patients. In contrast,endoscopic transaxillary augmentation allows the surgeon to divide the origin of the pectoral muscle under direct visualization, thereby effectively lowering the inframammary crease

138
Q

Elongation and laxity of which of the following structures are most likely to result in breast ptosis?
(A) Clavipectoral fascia
(B) Cooper’s ligaments
(C) Costoclavicular ligaments
(D) Superficial fascia of the breast
(E) Superficial fascia of the pectoralis muscle

A

(B) Cooper’s ligaments

139
Q

Anatomical cause of breast ptosis

A

Anatomically, ptosis is caused by disruption or elongation of Cooper’s ligaments, which are fibrous projections that arise from the breast tissue and fuse with the superficial fascia and dermis of the breast. These ligaments attach the breast parenchyma to the overlying skin.

140
Q

Cooper’s ligaments

A

Cooper’s ligaments are fibrous projections that arise from the breast tissue and fuse with the superficial fascia and dermis of the breast. These ligaments attach the breast parenchyma to the overlying skin.

141
Q

The clavipectoral fascia

A

The clavipectoral fascia covers the axilla and pectoralis minor muscle; this layer is encountered during axillary dissection. T

142
Q

The costoclavicular ligaments

A

The costoclavicular ligaments anchor the clavicle to the chest beneath the medial superior pole of the breast, but do not enter the breast parenchyma.

143
Q

The superficial fascia of the breast

A

The superficial fascia of the breast is a filmy, white layer of connective tissue located 2 to 15 mm deep to the skin. The deep layer of the superficial fascia envelopes the breast posteriorly.

144
Q

Plane between the superficial fascia of the breast and the deep fascia of the pectorals muscle

A

A loose areolar plane is present between the superficial fascia of the breast and the deep fascia of the pectoralis muscle, and facilitates removal of the breast from the pectoralis muscle during mastectomy.

145
Q

In women undergoing augmentation mammaplasty with saline-filled implants, which of the following techniques is most likely to decrease the longevity of the implant and lead to early rupture?
(A) Underfilling of the implants below the manufacturer’s recommended minimum
(B) Fillingof the implants to the manufacturer’s recommended minimum
(C) Filling of the implants to the volume between the manufacturer’s recommended minimum and maximum
(D) Filling of the implants to the manufacturer’s recommended maximum
(E) Overfilling of the implants above the manufacturer’s recommended maximum

A

(A) Underfilling of the implants below the manufacturer’s recommended minimum

146
Q

Which of the following techniques is indicated to preserve sensation to the nipple-areola complex in a patient undergoing augmentation mammaplasty?
(A) Avoiding periareolar incisions
(B) Avoiding sharp dissection near the clavicle
(C) Identifying and tagging of the sensory nerves as they exit the fascia
(D) Performing blunt dissection lateral to the lateral edge of the pectoralis muscle
(E) Positioning the implant subpectorally

A

(D) Performing blunt dissection lateral to the lateral edge of the pectoralis muscle

Performing blunt dissection lateral to the lateral edge of the pectoralis muscle only is indicated to preserve sensation to the nipple-areola complex.

147
Q

Surgical technique to preserve sensation to the nipple-areola complex during augmentation mammaplasty

A

The fourth and fifth anterolateral intercostal nerves perforate the fascia just lateral to the pectoralis muscle through the interdigitation of the serratus anterior muscle. By performing blunt dissection only lateral to the pectoralis muscle, these nerves are stretched but not cut. Although the stretching of sensory nerves may still result in loss of sensation, it is more likely to be temporary than if the nerves are cut sharply

148
Q

In a 50-year-old woman who underwent augmentation mammaplasty with silicone implants 12 years ago, a silicone granuloma is noted in the axillary region on clinical examination. Which of the following statements best characterizes this finding?
(A) Silicone granulomas are a frequent complication following augmentation mammaplasty or reconstruction with silicone implants
(B) Silicone granulomas indicate a link to the existence of implant-related systemic disease
(C) Silicone granulomas represent a common tissue response to foreign materials
(D) Surgical resection is rarely indicated

A

(C) Silicone granulomas represent a common tissue response to foreign materials

Although silicone granulomas are a well-recognized tissue response to the presence of foreign material, such as silicone, these granulomas are found only rarely in patients who have undergone augmentation mammaplasty or reconstruction with silicone gel breast implants. Any granulomas that are detected should be resected if they are symptomatic or of diagnostic concern.

149
Q
In patients with polymastia, accessory mammary structures are most frequently found at which of the following sites?
(A) Neck
(B) Axilla
(C) Thigh
(D) Buttock
(E) Vulva
A

(B) Axilla

150
Q

Which of the following factors is most critical in determining the need for breast contouring following removal of breast implants?
(A) Age of the patient
(B) Amount of breast tissue overlying the implant
(C) Degree of preoperative ptosis
(D) Size of the areola
(E) Size and position of the implant

A

(C) Degree of preoperative ptosis

151
Q

Most important factor / when to consider breast contouring after implant explantation

A

The degree of ptosis seen preoperatively is most important in determining the need for breast contouring following explantation. Because ptosis remains relatively unchanged following implant removal, contouring should be considered in women who have ptosis that is classified preoperatively as grade II or III.

152
Q

What determines the viability of performing breast contouring concomitantly with explantation

A

The thickness of residual breast parenchyma best determines the viability of performing breast contouring concomitantly with explantation. The breast tissue should have a minimum thickness of 4 cm to allow for vascularity of the overlying skin and of the separated glandular-nipple flap. This is best assessed by performing a breast pinch test superiorly and inferiorly.

153
Q

When is circumareolar mastopexy an option (in a patient who will undergo mastopexy)?

A

Circumareolar mastopexy is an option in women with areolae that are larger than 50 mm

154
Q

Which of the following findings is most likely in a patient with Poland syndrome?
(A) Anomalies of the feet
(B) Bilateral abnormalities of the ribs
(C) Breast hypertrophy
(D) Hypoplasia of the pectoralis major muscle
(E) Polythelia

A

(D) Hypoplasia of the pectoralis major muscle

Poland syndrome is a congenital anomaly that is characterized by unilateral aplasiaor hypoplasia of the pectoralis major muscle and adjacent musculoskeletal components.

155
Q
In a patient with breast implants, each of the following has been shown to interfere with screening mammography EXCEPT
(A) Baker III capsular contracture
(B) implant location
(C) implant size
(D) native breast volume
A

(C) implant size

156
Q

Edlund techniques / compression techniques

A

it is important for patients who have breast implants to undergo mammographic evaluation at specialized centers experienced at obtaining mammograms using either compression or displacement (Eklund) techniques, which maximize visualization of the breast parenchyma

157
Q

Factors shown to affect findings on mammography in women with breast implants

A

Several factors have been shown to affect the findings on mammography in women with breast implants. The positioning of the implant and the degree of associated capsular contracture have been known to influence the quantity of breast tissue that can be visualized. In addition, one study showed an increase in the amount of tissue that can be visualized postoperatively in a subset of women with small native breast volume.

The size of the implant has not been shown to affect the amount of breast tissue that can be visualized on mammography.

158
Q

In order to make the diagnosis of Poland’s syndrome,which of the following findings must be present?
(A) Absence of the nipple
(B) Absence of the sternal head of the pectoralis major muscle
(C) Brachysyndactyly
(D) Hypoplasia of the latissimus dorsi muscle
(E) Skeletal abnormalities of the chest wall

A

(B) Absence of the sternal head of the pectoralis major muscle

159
Q

A 24-year-old woman has worsening pain and swelling of the right breast 24 hours after undergoing subpectoral augmentation mammaplasty with smooth, round saline-filled implants. On physical examination, the right breast appears significantly larger and is more firm to palpation than the left breast. There are no signs of erythema or ecchymosis. Which of the following is the most appropriate next step in management?
(A) Observation
(B) Application of an external compression bandage
(C) Percutaneous needle aspiration
(D) Ultrasound-guided drainage
(E) Surgical exploration

A

(E) Surgical exploration

160
Q

Hematoma develops in what percent of patients undergoing breast augmentation?

A

Hematoma develops in 1% to 3% of patients who have undergone breast augmentation

161
Q

Timeline for hematoma development after breast augmentation

A

Hematomas can be seen as late as 14 days postoperatively

162
Q

Management of patient who has developed hematoma after breast augmentation

A

The most appropriate next step in management is prompt surgical exploration to evacuate the hematoma and ensure careful hemostasis. The implant can be replaced if there is no evidence of infection; the contralateral implant should only be removed if it is affected.

163
Q
Prior to breast augmentation, management of milky discharge in a regularly menstruating woman should include which of the following?
(A) Observation
(B) Massage
(C) Measurement of serum prolactin level
(D) Administration of antibiotics
(E) Ovarian biopsy
A

(C) Measurement of serum prolactin level

164
Q

Rate of breast discharge in women who have been pregnancy in the past

A

Although breast discharge is rare in regularly menstruatingwomen who have never been pregnant, it has been shown to occur in 25% of women who have been pregnant in the past.

165
Q

Evaluation of galactorrhea

A

Complete evaluation of galactorrhea should include measurement of the serum level of prolactin (a lactogenic hormone required for milk production), thyroid function studies to rule out hypothyroidism, and a history of all medications, as tricyclic antidepressants and fluoxetine have been shown to contribute to breast discharge.

166
Q

Prevalence of pituitary tumor in patients evaluated with galactorrhea

A

According to a series of four studies involving more than 500 patients with galactorrhea, a pituitary tumor was the underlying cause in 25%; in contrast, 50% of those studied had idiopathic causes.

167
Q
Which of the following is the most common complication of periareolar mastopexy?
(A) Dehiscence
(B) Excessive breast projection
(C) Nipple discharge
(D) Recurrent ptosis
(E) Widening of the areola
A

(E) Widening of the areola

Less common complications include dehiscence and recurrent ptosis. Excessive projection is rarely seen with periareolar mastopexy; flattened or globular breast shapes are more commonly reported.

168
Q

Most common complication of periareolar mastopexy

A

Widening of the areola

169
Q

Methods to prevent areolar dilation after periareolar mastopexy

A

Techniques developed to minimize the occurrence of areolar dilation include the use of nonresorbable purse-string sutures and creation of an excessively small areola at the time of surgery to compensate for postoperative widening.

170
Q

A 21-year-old woman desires surgical correction because her left breast has an abnormal appearance. On examination, the diameter of the left breast is more narrow at the base than at the midportion, and there is superior displacement of the inframammary fold. The areola is disproportionally enlarged, and the breast tissue appears to be herniating into the areola. The left cup size of her bra is 32B, and the right cup size is 32C. The right breast is normal. Which of the following is the most appropriate management?
(A) Right-sided vertical breast reduction
(B) Pedicled TRAM flap reconstruction of the left breast
(C) Wise-pattern breast reduction on the right with lowering of the inframammary fold
(D) Augmentation mammaplasty on the left using a saline-filled implant
(E) Augmentation mammaplasty on the left with radial scoring and areolar reduction

A

(E) Augmentation mammaplasty on the left with radial scoring and areolar reduction

171
Q

In a 21-year-old woman considering augmentation mammaplasty with saline-filled implants, which of the following is appropriate advice concerning potential complications of the procedure?
(A) Breast implants do not affect mammographic visualization of all breast tissue
(B) Capsular contracture requiring revision occurs in 2% of patients
(C) Infection is more common than hematoma
(D) Revision procedures are performed in 25% of patients within the first 10 years
(E) The risk for deflation is approximately 10% annually

A

(D) Revision procedures are performed in 25% of patients within the first 10 years

Even though specialized views are required for mammography screening in patients with breast implants, it is estimated that approximately 5% of the breast parenchyma is not fully visible on a mammogram. The two studies described above reported rates of significant capsular contracture ranging from 20% to 25%. Hematoma occurred in 3% of implant patients, but only 1% of patients developed infection. Deflation occurred in 1% of patients annually.

172
Q
Which of the following is the most common cutaneous branching pattern of the fourth intercostal nerve as it supplies innervation to the nipple-areola complex?
(A) Anterior
(B) Central
(C) Inferior
(D) Lateral
(E) Superior
A

(D) Lateral

173
Q

Course of lateral cutaneous intercostal branches to the nipple

A

In 93% of patients undergoing breast surgery, the lateral cutaneous branches coursed deeply within the pectoral fascia and reached the nipple from its posterior surface.

In contrast, 7% of patients undergoing breast dissection had lateral cutaneous branches coursing superficially within the subcutaneous fat, reaching the nipple from its lateral side.

174
Q

Central/inferior/cutaneous branches to the NAC

A

Anatomic studies of the intercostal nerves have failed to identify any central, inferior, or superior cutaneous branches to the nipple-areola complex.

175
Q

Course of the anterior cutaneous branches to the nipple

A

The anterior cutaneous branches course superficially within the subcutaneous tissue and terminate at the medial areolar border.

176
Q
Which of the following proteins has been implicated in the pathogenesis of breast implant capsule formation?
(A) Albumin
(B) Fibrinogen
(C) Complement
(D) IgG
A

(B) Fibrinogen

177
Q

What (intrinsic component) has been implicated in the pathogenesis of breast implant capsule formation?

A

The surface-bound protein fibrinogen has been implicated in the generation of inflammatory responses to biomaterials (ie, implants); early protein absorption of these biomaterials mediates the foreign body response.

178
Q

Silicone polymers are important biomaterials because they have which of the following characteristics?
(A) Biological inertness
(B) Hydrophilic nature
(C) Impermeability
(D) Resistance to contamination in the manufacturing process

A

(A) Biological inertness

179
Q

Why are silicones widely used for implantation?

A

Silicones such as polydimethylsiloxane are widely used materials for implantation because of their biocompatibility or biological inertness.

180
Q

Silicones and water

A

Because silicones are hydrophobic, not hydrophilic, water is repelled and the implanted materials will therefore not interact with enzymes or chemicals within the body.

181
Q

Silicones and tensile strength

A

Because these materials have relatively poor tensile strength, strict standards are imposed during the production of medical-grade silicone because of the propensity for contamination.

182
Q

A 36-year-old woman is being evaluated 17 years after undergoing augmentation mammaplasty with silicone gel implants. On examination, the implants are soft and minimally palpable; she reports no complications. This patient is at risk for which of the following?
(A) Implant rupture
(B) Increased silicon levels in breast milk
(C) Rheumatoid arthritis
(D) Scleroderma
(E) Silicone synovitis

A

(A) Implant rupture

183
Q
A 25-year-old woman who smokes cigarettes undergoes bilateral explantation of ruptured breast implants. On preoperative examination, she has severe ptosis with breast thickness of less than 4 cm; the nipple-areolar complex is positioned 5 cm below the inframammary crease. Which of the following surgical procedures would most effectively re-establish aesthetic breast contour?
(A) Delayed mastopexy
(B) Inframammary fold wedge excision
(C) Periareolar mastopexy
(D) Modified Kiel (vertical) mastopexy
(E) Wise pattern mastopexy
A

(A) Delayed mastopexy

Indications for this procedure include severe ptosis requiring nipple elevation of 4 cm, a breast mound smaller than 4 cm, and a significant history of smoking. Because this patient has many risk factors and moderate ptosis, requiring 2 cm to 4 cm of nipple repositioning, a two-stage procedure is recommended to reduce the risk for potential complications, including skin loss or compromise of the nipple-areolar complex.

Simultaneous breast contouring procedures should be avoided in these patients. The initial stage involves explantation and capsulectomy using an inframammary approach; elective mastopexy is then performed three months later.

184
Q

Indications / timing of delayed mastopexy after implant explantation

A

Indications for this procedure include severe ptosis requiring nipple elevation of 4 cm, a breast mound smaller than 4 cm, and a significant history of smoking.

Simultaneous breast contouring procedures should be avoided in these patients. The initial stage involves explantation and capsulectomy using an inframammary approach; elective mastopexy is then performed three months later.

185
Q

Inframammary fold wedge excision is recommended for:

A

Inframammary fold wedge excision is recommended for patients with pseudoptosis. This is defined as adequate breast volume and positioning of the nipple above the inframammary crease with a nipple-to-inframammary crease distance of greater than 6 cm. The wedge excision technique involves transposition of the inferior dermal parenchymal flap in order to increase breast projection.

186
Q

When can a modified Kiel mastopexy be performed?

A

A tension-free periareolar mastopexy can be performed to reposition the nipple if it lies more than 2 cm below the inframammary fold and has a diameter of less than 50 mm.

187
Q

When can a periareolar mastopexy be performed?

A

If the areolar diameter is greater than 50 mm and more than 2 cm of repositioning is required, a modified Kiel (vertical) mastopexy is recommended.

188
Q

When should a Wise pattern / similar mastopexy be performed?

A

Patients who require repositioning of 2 cm to 4 cm should undergo Wise pattern or a similar type of mastopexy

189
Q

Which of the following is the most accurate statement regarding the patient who has the findings shown in the above photograph? (Poland’s)
(A) This condition has been shown to initially develop at or around the third week of gestation
(B) This condition is typically familial
(C) Contralateral hand anomalies are associated
(D) Hypoplasia of the pectoralis major muscle is associated
(E) This patient is at increased risk for the development of carcinoma of the breast

A

(D) Hypoplasia of the pectoralis major muscle is associated

190
Q

Etiology of Poland’s syndrome

A

Poland’s syndrome typically occurs sporadically and its etiology is not fully understood; however, one theory implicates a vascular event occurring at or around the seventh week of gestation, when the developing hand lies close to the chest wall

191
Q

Which of the following findings are consistent with tuberous breast syndrome?
(A) Deficiency of the skin envelope, a decrease in vertical breast height, breast hypoplasia, and absence of the pectoralis major muscle
(B) Deficiency of the skin envelope, a decrease in vertical breast height, breast hypoplasia, and areolar hypertrophy
(C) Deficiency of the skin envelope, elongation of vertical breast height, breast hypertrophy, and absence of the pectoralis major muscle
(D) Redundancy of the skin envelope, a decrease in vertical breast height, breast hyperplasia, and absence of the pectoralis major muscle
(E) Redundancy of the skin envelope, elongation of vertical breast height, breast hyperplasia, and areolar hypertrophy

A

(B) Deficiency of the skin envelope, a decrease in vertical breast height, breast hypoplasia, and areolar hypertrophy

192
Q
A 45-year-old woman who underwent bilateral augmentation mammaplasty with silicone gel implants 20 years ago has developed capsular contracture involving one of her implants. She is concerned about the integrity of the implants. Ultrasonography suggests intracapsular rupture of the implant.Which of the following is the most appropriate next step in management?
(A) Observation
(B) Level-two ultrasonography
(C) Mammography
(D) MRI
(E) Surgery
A

(E) Surgery

In this woman and other symptomatic patients whose implants are more than ten years old, the high probability of true rupture (94%) after positive findings on ultrasonography obviates the need for any further diagnostic testing such as MRI.

193
Q

Probability of silicone implant rupture with negative ultrasound in an asymptomatic patient (< 10 yo)

A

If screening ultrasonography shows no rupture, the probability of rupture drops to 2.2%. No further work-up is necessary

194
Q

Probability of silicone implant (<10 yo) rupture with positive ultrasound in an asymptomatic patient

A

If ultrasonography suggests rupture, the relatively low probability (37.8%) of true rupture requires a confirmatory test using MRI

195
Q

Symptomatic patient after breast augmentation with silicone prosthesis which are < 10 yo:

W/positive ultrasound:
W/positive MRI

A

31%
w/ ultrasound: 80%
w/ MRI: 98%

196
Q

Symptomatic patient after breast augmentation with silicone prosthesis which are < 10 yo, w/positive ultrasound:

A

94%, obviating need for MRI

197
Q

Which of the following is most characteristic of an in vivo subglandular breast implant that was placed 10 years ago?
(A) Changes in the implant shell that may interfere with mammography
(B) Easier palpability resulting from increased stiffness of the implant shell
(C) Increased potential for the development of immune-related disorders
(D) Invasion of the implant shell by surrounding periprosthetic capsular tissue
(E) Loss of biomechanical shell strength when compared with preimplantation levels

A

(E) Loss of biomechanical shell strength when compared with preimplantation levels

198
Q

Dual plane augmentation

A

Placing implant under the pec major superiorly, but not inferiorly, to allow:

  • Preferential augmentation of the lower pole
  • Minimize flexion/contracture deformity when patients flex pec muscles
  • In ptotic breasts, prevent a ‘double chin’ where the breast tissue just seems to hang below an augmented upper pole
199
Q
A 24-year-old nulliparous woman comes to the office for augmentation mammaplasty. She currently wears a size 34B brassiere and wants her brassiere size to be increased to a D cup. She is a good candidate for subglandular placement of implants. Which of the following risks is decreased by the use of the textured silicone shell compared with the smooth silicone shell?
A) Capsular contracture
B) Hematoma
C) Prosthesis malposition
D) Rippling
E) Symmastia
A

A) Capsular contracture

Texturing of the implant surface has been shown to decrease the rate of capsular contracture when compared with smooth implants when the implants are placed in the subglandular position. The benefit of textured implants may not be present when the implants are placed in a submuscular pocket.

There is no difference in hematoma rates for textured versus smooth implants. Both symmastia and implant malposition are related to pocket dissection and not related to the type of implant placed. In the case of symmastia, the pockets have encroached upon the sternum and are close to each other or are touching. Implant malposition can be related to factors such as inadequate dissection of the pocket, or over-dissection of the pocket. Finally, some studies have demonstrated an increase in rippling with textured implant when compared with smooth implants. However, rippling may be more related to cohesiveness of the gel and fill volumes of the shell, because early reports of experience with the form-stable implant (Natrelle 410) seem to show decreased rates of rippling.

200
Q
A 49-year-old woman is evaluated because of a traumatic laceration of the right lower eyelid and cheek. Physical examination shows difficulty with eyelid closure, voluntary squinting, and animation. Which of the following branches of the facial nerve is most likely injured?
A) Buccal
B) Cervical
C) Marginal mandibular
D) Temporal
E) Zygomatic
A

E) Zygomatic

Anatomically, the orbicularis oculi muscle is divided into three segments: pretarsal, preseptal, and orbital. However, functionally, the orbicularis oculi muscle is divided into the medial inner canthal orbicularis and the extracanthal orbicularis. The medial inner canthal orbicularis is responsible for blinking, lower lid tone, and the pumping mechanism of the lacrimal system. Innervation to the inner canthal orbicularis is from the buccal branches of the facial nerve. The zygomatic branch of the facial nerve innervates the extracanthal orbicularis, which controls eyelid closure, voluntary squinting, and animation. The temporal, marginal mandibular, and cervical branches do not provide innervation to the orbicularis oculi muscle.

201
Q

Which of the following is the most common complication associated with “donut” mastopexy?
A) Boxy breast shape
B) Increased distance from nipple to inframammary fold
C) Loss of nipple sensation
D) Nipple necrosis
E) Widening of the areola

A

E) Widening of the areola

A common complication of the “donut” (circumareolar) mastopexy is widening of the areola. This can be minimized by using a Gore-Tex suture placed using the “wagon-wheel” technique and limiting the amount of skin resected to a 2:1 ratio of outside diameter to areolar diameter.

Boxy breast shape is associated with Wise pattern mastopexy. Nipple necrosis is associated with combined augmentation and mastopexy. Increased distance from the nipple to the inframammary fold is associated with vertical mastopexies in which the height of the medial and lateral pillars is too tall. Loss of nipple sensitivity is unusual because there is no parenchymal resection.

202
Q

A 27-year-old woman is evaluated because of pain 2 weeks after undergoing subglandular augmentation mammaplasty. She has no history of fever, chills, or drainage. Physical examination discloses a painful, tender cord in the inframammary region of the left breast. Which of the following is the most appropriate next step in management?
A) Administration of an antibiotic
B) Administration of an anticoagulant
C) Administration of an anti-inflammatory agent
D) Duplex ultrasonography
E) Removal of the implant

A

C) Administration of an anti-inflammatory agent

Mondor disease of the breast is a benign, self-limiting thrombophlebitis of the inframammary veins. Clinically, Mondor disease usually occurs 2 to 3 weeks postoperatively as a painful, tender cord within the superficial veins of the thoracoepigastric system. Management is observation and includes the use of warm, moist dressings and anti-inflammatory agents for symptomatic relief. The use of anticoagulation, antibiotics, or steroids is not indicated. Implant removal is not indicated in the absence of infection. Duplex ultrasonography is not required for management.

203
Q

A 65-year-old woman comes to the office 1 month before a scheduled mastopexy. Annual mammography shows a 1.5-cm mass in the upper outer quadrant. Core needle biopsy is performed. Pathologic examination of excised tissue identifies papilloma without atypia. Which of the following is the most appropriate next step in management?
A) Bilateral breast sonography
B) Excisional biopsy of needle-localized area
C) Repeat annual mammography in 12 months
D) Repeat mammography at 6-month intervals for 1 year
E) Stereotactic vacuum-assisted biopsy

A

B) Excisional biopsy of needle-localized area

Percutaneous biopsy methods are commonly accepted for the initial evaluation of clinically occult breast lesions, although certain nonmalignant lesions pose dilemmas with respect to the most appropriate clinical management. Papillary lesions of the breast can either be benign or malignant, although differentiation is radiologically difficult. Moreover, it is difficult for pathologists to reliably distinguish among benign, atypical, and malignant papillary lesions on the limited fragmented tissue specimens they receive after needle sampling.

Previous studies have demonstrated high rates of ductal carcinoma in situ (11%) in patients diagnosed with benign papillomas by needle biopsy and who subsequently underwent a surgical excision, although conflicting data suggest an extremely decreased rate of malignancy when histology is benign on needle biopsy.

The management of benign papillary lesions is somewhat controversial. Although conservative follow-up with either yearly mammogram or short-interval follow-up may be appropriate for certain patients diagnosed with benign papilloma, certain features of this patient’s lesion make conservative follow-up inappropriate. Sonographic follow-up in a 65-year-old woman with mature breast parenchyma and a solid mammographically detected mass would not provide much additional information, and a repeat percutaneous biopsy, whether core needle or vacuum-assisted, would also not be effective. Given the size of the lesion and the age of the patient, surgical excision is warranted despite the lack of atypia on needle biopsy. Benign papillomas tend to be smaller than 1 cm and centrally located, whereas malignant lesions are more often greater than 1.5 cm and are peripherally located.

204
Q

Benign vs malignant papillomas

A

Benign papillomas tend to be smaller than 1 cm and centrally located, whereas malignant lesions are more often greater than 1.5 cm and are peripherally located.

205
Q

A 28-year-old woman desires augmentation mammaplasty with silicone implants. Physical examination shows tuberous breast deformity with an elevated inframammary crease. Sternal notch to nipple distance is 21 cm bilaterally. Nipple to inframammary crease distance is 3.5 cm bilaterally. Periareolar mastopexy with 350-mL silicone implants is planned. Which of the following operative plans will most effectively minimize the likelihood of a double-bubble deformity?
A) Lower the inframammary crease by 3 cm
B) Perform radial release of the lower pole breast fascia
C) Place implants in subparenchymal pocket
D) Reinforce the inframammary crease with acellular dermal matrix
E) Use highly cohesive gel implants

A

B) Perform radial release of the lower pole breast fascia

The tuberous breast is a developmental deformity characterized by a constricted inframammary fold, short nipple to inframammary crease distance, and both horizontal and vertical deficiencies. The pathophysiology of the tuberous breast predisposes the patient to develop a double-bubble deformity. In this patient, the inframammary crease must be lowered to accommodate the implant and improve the vertical skin deficiency. Radial release of the lower pole breast fascia is done with either a cautery or a knife. Multiple radial incisions are made, thereby allowing the tight crease to expand and decrease the chance for a double-bubble deformity.

Lowering the crease is necessary but will increase the chances of a double-bubble deformity. Subparenchymal implant placement and use of highly cohesive gel implants may help but are not the essential procedures required. The use of acellular dermal matrix can help secure the position of the inframammary crease in a patient who develops a double-bubble deformity secondary to an inferior migration of the implant below the inframammary crease. This does not apply in the patient described.

206
Q
A 37-year-old woman comes to the clinic to be evaluated for augmentation mammaplasty to improve her breast shape. She is gravida 3, para 3, and breast-fed all of her children. On examination, she has decreased superior pole volume, and the distance from nipple to sternal notch is 28 cm. The nipple-areola complex is below the inframammary fold by 4 cm and is at the lower contour of the breast. Which of the following Regnault classifications of ptosis best describes these findings?
A) Grade I
B) Grade II
C) Grade III
D) Pseudoptosis
A

C) Grade III

The Regnault classification of breast ptosis is based on the relationship of the nipple to the inframammary fold (IMF) and to the lower contour of the gland.

Pseudoptosis is the not true ptosis. In this situation, the nipple is above the level of the IMF but the breast parenchyma has descended below the IMF.

Grade I is minor ptosis with the nipple at the level of the IMF and above the lower contour of the gland.

Grade II is moderate ptosis with the nipple below the level of the IMF and above the lower contour of the gland.

Grade III is major ptosis with the nipple below the level of the IMF and at the lower contour of the gland.

207
Q
A French woman, who underwent placement of Poly Implant Prothèse (PIP) gel implants in 2009, comes to the office for consultation because she had heard that the implants were filled with a nonmedical grade silicone. She reports that she has not had any problems with the implants, but would like to know the implications of retaining the implants and whether she should have them removed. This patient should be told that she is at increased risk for which of the following complications if she retains the implants?
A) Breast cancer
B) Cytotoxicity
C) Heavy metal poisoning
D) Implant rupture
E) Siloxane poisoning
A

D) Implant rupture

The final report, in conjunction with the Department of Health in Australia, has shown a 2 to 6 times increased rupture rate in Poly Implant Prothèse (PIP) implants, which is detectable within 5 years of implantation. Increased levels of siloxane have been detected, but are not considered a health risk. No organic impurities have been detected and platinum levels are decreased in PIP gel compared with medical grade silicone. There is no increased breast cancer risk and no evidence of cytotoxicity. In the light of the increased rupture rate and the nonmedical grade nature of PIP silicone gel, the following recommendations were made:

all providers of breast implant surgery should contact any women who have or may have PIP implants, if they have not already done so, and offer them a specialist consultation and any appropriate investigation to determine if the implants are still intact;
if the original provider is unable or unwilling to do this, a woman should seek referral through her general practitioner to an appropriate specialist;
if there is any sign of rupture, she should be offered an explantation;
if the implants still appear to be intact, she should be offered the opportunity to discuss with her specialist the best way forward;
if, in the light of this advice a woman decides with her specialist that, in her individual circumstances, she wishes to have her implants removed, her health care provider should support her in carrying out this surgery. Where her original provider is unable or unwilling to help, the NHS will remove, but not normally replace, the implants;
if a woman decides not to seek early explantation, she should be offered annual follow up in line with the advice issued by the specialty surgical associations in January 2012. Women who make this choice should be encouraged to consult their doctor if they notice any signs of tenderness or pain, or swollen lymph glands in or around their breasts or armpits, which may indicate a rupture. At the first signs of rupture, they should be offered removal of the implants.

208
Q
A 35-year-old woman comes to the office for consultation because she is dissatisfied with the appearance of her “deflated” and “saggy” breasts. Augmentation/mastopexy is planned. Compared with placement of the implant in the subglandular position, placement of the implant in the subpectoral space will preserve blood supply to the breast tissue and skin through which of the following arteries?
A) Internal thoracic
B) Lateral thoracic
C) Superficial superior epigastric
D) Thoracoacromial
E) Thoracodorsal
A

D) Thoracoacromial

The perfusion of the nipple-areola complex is a major concern during breast procedures involving periareolar and intraparenchymal incisions. The nipple-areola complex has a very rich and overlapping perfusion through multiple sources. This fact allows the design of various pedicles to carry the nipple and areola with different techniques. The blood supply through the internal thoracic vessels reaches the breast, nipple, and areola through the intercostal perforators, which may be divided during both subpectoral and subglandular implant placement.

The location of the implant deep or superficial to the pectoralis muscle will not change the perfusion through the superficial epigastric vessels. The same is true for the blood supply through the lateral thoracic vessels. However, the flow through the thoracoacromial vessels to the breast parenchyma will be preserved by placement of the implant deep to the pectoralis muscle. Creation of a subglandular pocket above the muscle will interrupt the collaterals from the thoracoacromial vessels through the muscle to the parenchyma.

The thoracodorsal artery is not a major source of blood supply to the breast and the position of the implant will not affect it.

209
Q

A 32-year-old woman is scheduled to undergo augmentation mammaplasty with highly cohesive, anatomically shaped, silicone-filled breast implants. She asks the surgeon about postoperative monitoring for implant rupture. This patient should be counseled that, according to FDA recommendations, postoperative monitoring for rupture most appropriately includes which of the following?
A) Manual examination 3 years postoperatively, then annually thereafter
B) MRI screening 2 years postoperatively, then every 3 years thereafter
C) MRI screening 3 years postoperatively, then every 2 years thereafter
D) Ultrasonography screening 2 years postoperatively, then every 3 years thereafter
E) Ultrasonography screening 3 years postoperatively, then every 2 years thereafter

A

C) MRI screening 3 years postoperatively, then every 2 years thereafter

Diagnosis of rupture is difficult by physical examination alone, which is why the majority of ruptures are silent. Subsequent MRI screening for silent rupture is recommended initially 3 years postoperatively, then every 2 years thereafter.

Highly cohesive, anatomically shaped, silicone-filled breast implants combine the “gummy bear” silicone with an anatomical shape, in which inferior pole projection is higher than the superior pole projection. In studies of Allergan’s Natrelle 410 breast implants (the “Pivotal Study,” the 410 Swedish MRI study, and the 410 European MRI study) approximately 3 in 100 women had silent ruptures.

Cohesive gel is still subject to rupture, because rupture occurs when the shell fails. In cohesive implants, however, as opposed to noncohesive implants, the rupture rarely becomes extracapsular.

210
Q
A 45-year-old woman comes to the office 10 years after undergoing subglandular implantation of textured silicone implants for augmentation mammaplasty. Physical examination shows swelling of the left breast. She is concerned about cancer. Increased incidence of which of the following malignancies is associated with breast implants?
A) Acute myeloid leukemia
B) Anaplastic large cell lymphoma
C) Angiosarcoma
D) Infiltrating ductal carcinoma
E) Malignant fibrous histiocytoma
A

B) Anaplastic large cell lymphoma

Several reports have suggested an association between breast implants and anaplastic large cell lymphoma (ALCL), which is an extremely rare malignancy. In these cases, ALCL has usually occurred several years after implantation as swelling or a mass around the implant and is often associated with a periprosthetic seroma. Treatments have included capsulectomy with implant removal and chemotherapy and/or radiation therapy, though there is no defined consensus regimen. Despite evidence of an increased risk of ALCL in breast implant patients, the absolute risk remains extremely low.

Several large epidemiologic studies have demonstrated a similar or lower incidence of breast cancer (infiltrating ductal carcinoma) among patients who have undergone prosthetic augmentation mammaplasty surgery compared with those who have not. Most cases of ALCL have been in textured implants.

Angiosarcoma and malignant fibrous histiocytoma are two sarcomas that may arise in the breast. Angiosarcoma may be caused by radiation therapy for breast cancer. Neither of these sarcomas has been associated with breast implants.

Acute myeloid leukemia may be associated with radiation treatment to the breast but has not been associated with breast implants.

211
Q

A 33-year-old woman with no family history of breast cancer undergoes bilateral augmentation mammaplasty with 300 mL of autologous fat per breast. Six months later, she has onset of pain in the right breast. Mammography shows linear clustered microcalcifications in the lower inner quadrant of the right breast, small lipid cysts bilaterally with scattered dystrophic rod-like calcifications in the upper outer quadrants bilaterally, and heterogeneity of the pectoral muscles. Which of the following is the most appropriate next step in management?
A) Baseline mammography between ages 35 and 40 and yearly thereafter
B) Core needle biopsy of the bilateral upper outer quadrants
C) Core needle biopsy of the right lower inner quadrant
D) Repeat mammography at 6 months and 12 months
E) Repeat mammography in 1 year

A

C) Core needle biopsy of the right lower inner quadrant

Augmentation mammaplasty with autologous fat transfer has become an increasingly popular option for patients desiring modest volumetric improvement. Despite its popularity, there is still some concern regarding its safety and efficacy. ASPS offered guidelines on fat grafting for reconstructive procedures of the breast in 2009. However, caution is recommended in the setting of cosmetic procedures because the impact on radiologic changes in follow-up is still uncertain to date.

Fat necrosis is a nonspecific histologic finding most commonly resulting from surgery, trauma, or radiation therapy. It is common after fat transfer procedures, though often is clinically occult, and detected through follow-up mammography. The mammographic images of fat necrosis range from lipid cysts to findings that are suspected for malignancy such as clustered microcalcifications or spiculated masses. The most frequent mammographic finding in the breast parenchyma after augmentation mammaplasty with fat transfer is bilateral scattered microcalcifications followed by radiolucent oil cysts with or without microcalcification. Microcalcifications represent an evolution in the mammographic appearance of fat necrosis and are usually not present in early postoperative screening, but rather are a relatively late finding that is present months to years after the inciting trauma.

It is imperative that radiologists distinguish between benign and suspected microcalcifications in order to minimize the number of postoperative biopsies and frequent follow-up imaging. Although round, spherical, punctuate, and diffusely scattered calcifications are typical of benign processes, cluster, branching microcalcifications can be indicative of a malignant process and should be worked up. For this 33-year-old patient with no baseline mammography and a suspected lesion within 6 months of the procedure, routine or short-interval mammographic screening is not appropriate. A biopsy of the suspected area is required, and this patient should undergo a core needle biopsy of the clustered microcalcifications of the right breast, while the more benign-appearing calcifications within the upper outer quadrants can be observed.

212
Q
A 35-year-old woman with tuberous breast deformity is scheduled to undergo augmentation/mastopexy. A smooth, round, cohesive gel implant will be used. This patient is at higher risk for which of the following complications when compared with augmentation/mastopexy performed on a patient without a tuberous breast?
A) Capsule contracture
B) Double bubble
C) Hematoma
D) Nipple-areola depigmentation
E) Rippling
A

B) Double bubble

The classic features of a tuberous breast deformity include a constricted base with a high inframammary crease and herniation of breast parenchyma into the nipple-areola complex producing a large-diameter areola. Variable extent of micromastia is associated as well as breast asymmetry. When a patient has a high and tight inframammary crease, this crease must be released to accommodate an implant and allow correction of the deformity. If this native crease does not fully expand, then a double bubble will occur. Over time, the lower pole skin stretches in response to the implant and this double bubble often improves spontaneously. The incidence of capsule contracture, hematoma, nipple-areola depigmentation, and rippling should be similar to a patient who undergoes periareolar augmentation/mastopexy without a tuberous breast.

213
Q

A 28-year-old woman is scheduled to undergo vertical mastopexy. She has no history of previous breast surgery. A superior pedicle technique is planned. Which of the following is the dominant blood supply for this pedicle?
A) Deep branches of the internal mammary artery from the fourth interspace
B) Deep branches of the internal mammary artery from the fifth interspace
C) Superficial branches of the internal mammary artery from the second interspace
D) Superficial branches of the internal mammary artery from the fourth interspace
E) Superficial branches of the lateral thoracic artery

A

C) Superficial branches of the internal mammary artery from the second interspace

The breast receives its arterial blood supply from multiple sources, and this fact is used to design multiple pedicles for the nipple-areola complex that can work reliably for both mastopexy and reduction mammaplasty procedures.

The superior pedicle receives its arterial blood supply primarily from the internal mammary branch from the second interspace. It is usually about 1 to 2 cm below the surface of the skin just medial to the breast meridian as it approaches the areola and may be localized with a handheld Doppler device during preoperative planning.

The inferior pedicle and central pedicle designs are primarily supplied by branches of the internal mammary system from the fourth interspace. Additionally, there is some accessory input from the intercostal branches at the level of the inframammary fold with the inferior pedicle design. These secondary vessels are typically interrupted in a central pedicle operation.

The medial pedicle design receives its arterial input mainly from the third superficial branch of the internal mammary artery. This vessel may be damaged by previous augmentation mammaplasty.
The lateral pedicle design receives its arterial supply from superficial branches of the lateral thoracic artery.

214
Q

Superior medical breast surgery: Blood supply

A

The superior pedicle receives its arterial blood supply primarily from the internal mammary branch from the second interspace. It is usually about 1 to 2 cm below the surface of the skin just medial to the breast meridian as it approaches the areola and may be localized with a handheld Doppler device during preoperative planning.

215
Q

Inferior medical breast surgery: Blood supply

A

The inferior pedicle and central pedicle designs are primarily supplied by branches of the internal mammary system from the fourth interspace. Additionally, there is some accessory input from the intercostal branches at the level of the inframammary fold with the inferior pedicle design. These secondary vessels are typically interrupted in a central pedicle operation.

216
Q

Medial medical breast surgery: Blood supply

A

The medial pedicle design receives its arterial input mainly from the third superficial branch of the internal mammary artery. This vessel may be damaged by previous augmentation mammaplasty.

217
Q

Lateral medical breast surgery: Blood supply

A

The lateral pedicle design receives its arterial supply from superficial branches of the lateral thoracic artery.

218
Q

An otherwise healthy 40-year-old woman comes to the office for augmentation mammaplasty. Mammography 6 months ago showed no abnormalities. Family history is negative for breast cancer. She wants to know if silicone gel implants are safe and what she should do after the procedure to monitor the implant for evidence of rupture. According to the current federal guidelines, which of the following is the most appropriate recommendation to this patient regarding surveillance?
A) MRI 3 years after implantation and every 2 years thereafter
B) MRI every 10 years
C) MRI if symptoms such as chronic myalgia and fatigue develop
D) Yearly mammograms
E) Yearly MRI

A

A) MRI 3 years after implantation and every 2 years thereafter

Evidence-based data to confirm the validity of screening patients with silicone implants are lacking. In 2011, the FDA issued recommendations for physicians on the use of silicone gel-filled implants. Recommendations included providing copies of educational brochures, giving appropriate informed consent, maintaining medical vigilance, and reporting adverse events. It also suggested that patients undergoing augmentation mammaplasty get an MRI 3 years after implant placement and every 2 years thereafter. The purpose of these recommendations is not to replace routine cancer surveillance.

219
Q
A 30-year-old woman comes to the office for augmentation mammaplasty and mastopexy after a 50-lb (23-kg) weight loss. She wears a size 38B brassiere. Physical examination shows grade II ptosis and a sternal notch to nipple distance of 26 cm bilaterally. Simultaneous augmentation mammaplasty with short-T mastopexy using smooth saline-filled breast implants that will be implanted in a dual-plane configuration through an inframammary incision is planned. Which of the following factors puts this patient at highest risk for reoperation?
A) Inframammary implant insertion route
B) Presence of breast ptosis
C) Use of drains
D) Use of saline implants
E) Use of smooth-walled implants
A

B) Presence of breast ptosis

It has long been realized that combination augmentation mammaplasty operations are more difficult and have a higher revision rate than either operation alone. A recent review of 177 primary augmentation mammaplasty cases found that, of the factors listed, preexisting breast ptosis and simultaneous mastopexy were both linked to a higher rate of reoperation when possible contributing factors were statistically analyzed. Furthermore, increasing grades of breast ptosis were linked with increasingly higher reoperation rates.

Although incision site for augmentation mammaplasty has been markedly linked to the rates of capsular contracture, inframammary incisions have been shown in at least two studies to date to have the lowest rate of capsule formation, with periareolar and transaxillary incisions showing 5 to 10 times higher rates of capsule-related complications.

220
Q
A 30-year-old woman comes to the office because of a 3-week history of unilateral swelling of the left breast. She underwent subglandular placement of textured silicone breast implants 4 years ago. She has had no trauma, fevers, or chills. A 1-week course of an oral antibiotic prescribed by her family physician has failed to resolve the swelling. On physical examination, the left breast is 300 to 400 mL larger than the right breast. No other abnormalities are noted. Ultrasonography report shows seroma and results are negative for hematoma or mass. Which of the following is the most likely diagnosis in this patient?
A) Anaplastic large cell lymphoma
B) Double capsule phenomenon
C) Giant fibroadenoma of the breast
D) Hematoma due to capsule tear
E) Periprosthetic abscess
A

B) Double capsule phenomenon

The combination of late-onset swelling without signs of periprosthetic infection (fever, cellulitis), no history of trauma, and a negative ultrasonography suggests late-onset seroma, as can occur with a double capsule phenomenon. Late seromas occur as a complication in about 1% of reported breast implant series. This issue seems to be more common in the setting of textured implants, particularly those implants manufactured with an aggressive texturing process. At surgery, a capsule layer is seen lining the pocket, which often contains a substantial volume of serosangineous seroma fluid and a textured implant coated in a tight second capsule at the center of the pocket. Double capsule has been reported in both the subglandular and submuscular positions. A giant fibroadenoma of the breast would have a dominant mass, distortion of the breast shape, and would be visible on ultrasonography. Abscess would be likely to occur with fever, chills, and cellulitis of the breast. Hematoma of this size would be likely to have a history of trauma, breast pain, and external bruising. Although anaplastic large cell lymphoma is a possibility in the differential of late-onset seromas, it is a rare disorder. Seroma fluid, obtained either by ultrasound-guided aspiration or at the time of open surgery, should be sent for cytologic examination and immunohistochemistry to rule out this rare possibility.

221
Q

A 53-year-old woman comes to the office for evaluation of breast asymmetry. Reduction of the left breast and augmentation of the right breast with implant and autologous fat transfer are planned. She is concerned about fat injection and cancer risk. Which of the following is the most appropriate response regarding mammographic changes after fat transfer?
A) Calcifications warranting biopsy are more likely on the fat transfer side
B) Calcifications warranting biopsy are more likely on the reduction side
C) Masses requiring biopsy are more likely on the reduction side
D) Scarring will be decreased on the reduction side
E) There are no differences between mammographic findings in fat transfer and reduction

A

C) Masses requiring biopsy are more likely on the reduction side

Fat transfer to the breast remains a controversial procedure. There are some concerns about the oncologic safety of fat transfer, and for this reason some authors do not recommend fat transfer in patients with a history of cancer. Another concern about fat transfer is the potential difficulty in screening for malignancy. Rubin, et al. compared mammographic changes after fat transfer with changes after reduction mammaplasty. In this blinded study, radiologists reviewed pre- and postoperative mammograms of patients who had undergone augmentation and fat transfer and reduction mammaplasty. In the reduction cohort, masses requiring biopsy and scarring were more common; other abnormalities, including oil cysts, benign calcifications, and calcifications requiring biopsy showed no differences between the groups.

222
Q

A 53-year-old woman comes to the office because of unilateral swelling of the breast 5 years after undergoing subglandular augmentation mammaplasty. A diagnosis of anaplastic large T-cell lymphoma (ALCL) is established. Which of the following is most likely to represent the progression of this patient’s disease when compared with a patient who has ALCL but no breast prostheses?
A) A more aggressive clinical course and a poorer prognosis
B) A more aggressive clinical course but a more favorable prognosis
C) A more indolent clinical course and a more favorable prognosis
D) A more indolent clinical course but a poorer prognosis
E) The same clinical course and prognosis

A

C) A more indolent clinical course and a more favorable prognosis

Anaplastic large T-cell lymphoma (ALCL) is a rare (1 per million) non-Hodgkin lymphoma that has been reported in women with and without breast prostheses. However, increasing case reports suggest an association with breast prostheses, although direct causation has not been established. ALCL associated with breast prostheses has malignant cells infiltrating the periprosthetic capsule or in the periprosthetic fluid collection. It is associated with both silicone- and saline-filled prostheses and seen in patients who have had prostheses for augmentation mammaplasty as well as breast reconstruction. Although the cytology is the same between ALCL associated with and without breast prostheses, ALCL that develops around prostheses tend to have an indolent clinical course and favorable prognosis when compared with systemic ALCL.

223
Q

A 49-year-old woman is scheduled to undergo subglandular augmentation mammaplasty with silicone prostheses. During the preoperative discussion, the patient asks about postoperative complications with silicone versus saline prostheses. Which of the following is a disadvantage of using silicone in this patient?
A) Their rupture results in an obvious decrease in breast size
B) They are more likely to result in invasive breast cancer
C) They can obscure breast tissue on mammagraphy
D) They may show more rippling

A

C) They can obscure breast tissue on mammagraphy

Silicone prostheses are radiopaque on mammography. Therefore, when placed in the subglandular position, a small percentage of breast tissue is obscured on mammography. Breast prostheses made completely of or in part with silicone have not been shown to cause a delay in detection of breast cancer. Women with breast prostheses are not more likely to develop breast cancer. Women with breast prostheses who have developed breast cancer are not diagnosed at a more advanced stage and do not have a worse prognosis or survival when compared with women without prostheses. Silicone prostheses are less likely to show superior pole rippling when compared with saline prostheses. If a saline prosthesis ruptures, the saline tends to become absorbed by the body, resulting in an obvious decrease in breast size after a few days. When silicone prostheses rupture, the silicone may remain intracapsular. These ruptures may change the breast shape slightly but usually do not change the size and are often subclinical

224
Q
A 33-year-old woman comes to the office for consultation because she is dissatisfied with the "sagging" appearance of her breasts. Examination shows grade II ptosis and loss of fullness in the upper pole. A vertical mastopexy is planned. The most common medial innervation to the nipple-areola complex is the anterior cutaneous branches of which of the following intercostal nerves?
A) Second and third
B) Third and fourth
C) Fourth and fifth
D) Fifth and sixth
E) Sixth and seventh
A

B) Third and fourth

The most common medial innervation of the nipple-areola complex is mainly 57% from the anterior cutaneous branches of the third and fourth intercostal nerves. The third intercostal nerve accounts for 21.4%. They always reach the areolar edge between 8 and 11 o’clock on the left and 1 and 4 o’clock on the right. The nerve innervation to the nipple-areola complex is important in planning different incisions around the areola in both reduction mammaplasty and mastopexy.

225
Q

A 25-year-old woman is considering augmentation mammaplasty with silicone prostheses. The patient asks about the associated risks of developing connective tissue disease. Which of the following risk assessments is most accurate in this patient?
A)Increased risk of extracapsular leak only
B)Increased risk of intra- and extracapsular leak
C)Increased risk only if the silicone migrates to the lymph node
D)Increased risk only in the pre-1990 prostheses
E)No increased risk

A

E)No increased risk

Concern regarding an association between silicone breast prostheses and connective tissue disease was raised in the 1980s and early 1990s, eventually leading to the US Food and Drug Administration (FDA) moratorium of the use of silicone breast prostheses in augmentation mammaplasty. Since then, multiple cohort studies and case control studies in Europe and North America have failed to determine a causative association between silicone breast prostheses and any traditional or atypical connective tissue diseases.

226
Q

A 23-year-old woman comes to the office for consultation regarding surgical correction of a tuberous breast deformity. On physical examination, which of the following characteristics is most likely in this patient?
A) Absence of the sternal head of the pectoralis muscle
B) Effacement of the inframammary fold
C) Grade III ptosis of the nipple-areola complex
D) Herniation of breast tissue into the nipple-areola complex
E) Macromastia

A

D) Herniation of breast tissue into the nipple-areola complex

Physical examination of a tuberous breast would show herniation of the nipple-areola complex. A constricted inframammary fold, rather than an effaced inframammary fold, is often associated with tuberous breast deformity. Macromastia and/or grade III ptosis of the nipple-areola complex are not standard components of tuberous breast deformity. Absence of the sternal head of the pectoralis muscle is a characteristic feature of Poland syndrome.

227
Q

A 24-year-old woman with bilateral micromastia comes for consultation regarding augmentation mammaplasty. The patient says she would like her breasts to be “as big as possible.” On examination, which of the following is the most important factor in determining the maximum acceptable prosthesis size for this patient?
A) Breast base width
B) Diameter of the areola
C) Grade of nipple-areola ptosis
D) Maximum manufactured prosthesis volume
E) Pectoralis muscle height-to-prosthesis height ratio

A

A) Breast base width

The most important factor in determining the maximum acceptable prosthesis size in this patient is breast base width. Grade of nipple-areola ptosis, areola diameter, maximum manufactured prosthesis volume, and pectoralis height may all impact overall appearance of the breast but do not have an impact on breast prosthesis size choice.

228
Q

A 35-year-old woman comes to the office with her boyfriend for consultation regarding augmentation mammaplasty. She currently wears a size 34B brassiere and is considering having her brassiere size increased to a D cup. She says she is happy with the way she looks in clothes, but the boyfriend indicates he would like to see a little more cleavage when she is in a swimsuit. History includes liposuction of her lateral thighs 6 months ago by a local dermatologist; she was satisfied with the result. She has also had injection of botulinum toxin type A to the glabella 3 times in the last year. Which of the following is the best reason to refuse performing the procedure for this patient?
A) The patient may be being pushed into surgery
B) The patient may be a “surgiholic”
C) The patient may have body dysmorphic disorder
D) The patient may have a personality disorder
E) The patient may have unrealistic expectations

A

A) The patient may be being pushed into surgery

Most aesthetic surgeons and mental health professionals agree that patients who exhibit even mild signs of psychiatric problems are not good candidates for aesthetic surgery. Many patients present without obvious signs of problems and are unfortunately discovered when postoperative problems arise. However, there are certain groups of patients with easily identifiable characteristics that constitute a red flag: those who are pushed into surgery by others, those with whom you are incompatible, the ?surgiholic? with a long past surgical history, those facing marital or familial disapproval, those with body dysmorphic disorder, the overly demanding patient, and those with unrealistic expectations.

229
Q

A 25-year-old woman comes to the office because of a 1-week history of erythema and clear drainage from the right breast 6 weeks after undergoing bilateral augmentation mammaplasty. She is afebrile and her vital signs are within normal limits. The drainage from the breast is sent for cultures. Broad-spectrum antibiotics are administered, but no improvement is noted over the next 48 hours. Surgical debridement and explantation of the prostheses are performed. After 7 days, cultures grow Mycobacterium fortuitum. Which of the following is the most appropriate next step?
A) Administration of ciprofloxacin and trimethoprim-sulfamethoxazole for 6 weeks
B) Administration of ciprofloxacin and trimethoprim-sulfamethoxazole for 6 months
C) Administration of isoniazid, rifampicin, and pyrazinamide for 6 weeks
D) Administration of isoniazid, rifampicin, and pyrazinamide for 6 months
E) No antibiotic therapy is needed because the infected prostheses have been removed

A

B) Administration of ciprofloxacin and trimethoprim-sulfamethoxazole for 6 months

The most appropriate next step in management is to initiate a 6-month course of ciprofloxacin and trimethoprim-sulfamethoxazole (Bactrim). Mycobacterium fortuitum is an atypical, nontuberculous mycobacterium (NTM), and it is one of the most common causes of NTM soft-tissue infections. It occurs most commonly in the presence of foreign bodies, such as breast prostheses. The incidence of these opportunistic infections has increased over the years. NTM infections can be more indolent and manifest weeks, or even months, following surgery. They occur most commonly with erythema, swelling, and clear drainage, although purulence may be seen. Fever may be absent. On surgical exploration, exuberant granulation tissue and turbid, odorless fluid are often noted. Routine Gram stains and cultures are usually negative. Therefore, it is imperative to request acid-fast bacilli staining and mycobacterial cultures if suspicion of NTM infection is high. Removal of the prosthesis and thorough debridement of the periprosthetic space, followed by long-term (3 to 6 months) antibiotic therapy, is required to treat this infection. Culture sensitivities should guide the antibiotic regimen, but ciprofloxacin, trimethoprim-sulfamethoxazole (Bactrim), clarithromycin, and doxycycline are used commonly for treatment. Reimplantation of the prosthesis should not be considered for a period of at least 6 months.

Isoniazid, rifampicin, and pyrazinamide are standard antibiotics used to treat tuberculosis caused by Mycobacterium tuberculosis, not atypical mycobacteria. Although removal of the affected prosthesis is required, long-term antibiotic therapy is an essential part of the treatment.

230
Q
A 43-year-old woman comes to the office for consultation regarding augmentation mammaplasty. She has never had any lumps or nipple discharge from her breasts, and has no family history of breast cancer. After discussion, she chooses saline prostheses. She is concerned about breast cancer and inquires about screening. Which of the following screening studies is most appropriate for this patient after augmentation?
A) CT scan
B) Mammography
C) MRI
D) Positron emission tomography
E) Ultrasonography
A

B) Mammography

Current recommendations for breast cancer screening in women with augmentation mammaplasty include mammography with Eklund views. In the Eklund technique, the prosthesis is pushed back against the chest wall, and the breast tissue is pulled forward and around the prosthesis. The use of this technique increases the sensitivity of mammography for breast cancer. Breast prostheses may affect the visualization of breast tissue, and it has been suggested that diagnostic mammography be obtained instead of screening mammography, even for the asymptomatic patient.

CT scanning has been studied for the evaluation of the breast but is not routinely used as a tool for breast imaging. MRI is recommended for the evaluation of a ruptured silicone prosthesis. The technique has high sensitivity, but lower specificity and high cost. It is not recommended as a screening tool for breast cancer in the general population at this time, but it may play a role in the high-risk patient.

Positron emission tomography is not used as a screening test for breast cancer. It is often used as an adjunct in patients diagnosed with breast cancer to determine if the cancer has spread to the lymph nodes or other parts of the body. Ultrasonography may be used for screening but is not recommended because it is very operator dependent. It will often be used as an adjunct to mammography in screening or if a suspected lesion is found.

231
Q
A 50-year-old woman comes to the office for consultation about improving the appearance of her "saggy" breasts. She has lost 100 lb (45 kg) during the past 18 months by diet. Photographs are shown. Physical examination shows breast deflation and marked ptosis. A Wise pattern mastopexy with augmentation mammaplasty is planned. Which of the following arteries is most likely to provide circulation to the breast gland and nipple during submuscular augmentation in this patient?
A)Intercostal
B)Pectoral
C)Superior epigastric
D)Thoracoacromial
E)Thoracodorsal
A

D)Thoracoacromial

The thoracoacromial artery and vein travel just deep to the pectoralis major muscle, supplying circulation to the overlying breast tissue and skin. Subglandular augmentation mammaplasty disrupts the connection between the thoracoacromial vessels and the overlying breast. This leads to a higher risk of wound-healing complications when placing the prosthesis in the subglandular plane. The submuscular plane of dissection maintains the connection between the thoracoacromial vessel and overlying breast and skin, allowing better potential healing.

Intercostal arteries are multiple and are not completely disconnected with either subglandular or subpectoral augmentation mammaplasty.

The superior epigastric artery provides circulation to the rectus abdominis muscle and abdomen. This artery would be injured with the mastopexy procedure.

The thoracodorsal artery supplies the latissimus dorsi muscle and not the chest.

232
Q
A 35-year-old woman comes for consultation regarding breast prosthesis removal because she is concerned about her risk of cancer. Specifically, she has read about anaplastic large cell lymphoma in women with breast prostheses. She underwent augmentation mammaplasty with saline breast prostheses 5 years ago. Physical examination shows absence of contracture and satisfactory position. Which of the following is the most appropriate next step in management?
A) Complete blood cell count
B) Evaluation by a hematologist
C) MRI of the breasts
D) Prosthesis removal
E) Reassurance
A

E) Reassurance

The US Food and Drug Administration (FDA) searched its adverse event reporting systems for reports received between January 1, 1995 and December 1, 2010, including information submitted by manufacturers as part of their required post-approval studies. This search identified 17 reports of possible anaplastic large cell lymphoma (ALCL) in women with breast prostheses. Although ALCL is extremely rare, the FDA believes that women with breast prostheses may have a very small but increased risk of developing this disease in the scar capsule adjacent to the prosthesis. Based on available information, it is not possible to confirm with statistical certainty that breast protheses cause ALCL. Currently, it is not possible to identify a type of prosthesis (silicone gel versus saline) or a reason for implantation (reconstruction versus aesthetic augmentation) associated with a smaller or greater risk.

When ALCL occurs, it has been most often identified in patients undergoing prosthesis revision procedures for late-onset, persistent seroma. Because it is so rare and most often identified in patients with late onset of symptoms such as pain, lumps, swelling, or asymmetry, it is unlikely that increased screening of asymptomatic patients would change their clinical outcomes. The FDA does not recommend prophylactic breast prosthesis removal in patients without symptoms or other abnormalities.

A patient with suspected ALCL should be referred to an appropriate specialist for evaluation. When testing for ALCL, fresh seroma fluid and representative portions of the capsule should be collected and sent for pathology tests to rule out ALCL. Diagnostic evaluation should include cytologic evaluation of seroma fluid with Wright-Giemsa–stained smears and cell block immunohistochemistry testing for cluster of differentiation and anaplastic lymphoma kinase markers. Any confirmed cases of ALCL in women with breast prostheses must be reported to the FDA.

233
Q

A 45-year-old woman comes for evaluation 1 year after undergoing vertical mastopexy without placement of prostheses because she thinks her breasts have started to sag. An increase in which of the following breast dimensions has most likely occurred since the patient’s last visit?
A) Breast base diameter
B) Nipple to inframammary crease
C) Nipple-areola diameter
D) Suprasternal notch to inframammary crease
E) Suprasternal notch to nipple

A

B) Nipple to inframammary crease

The nipple-to-inframammary crease dimension is most likely to increase over time. This leads to pseudoptosis (bottoming out) and the appearance of a sagging breast. Pseudoptosis occurs when the breast gland migrates lower than the inframammary crease while the nipple stays in normal position. It is essential that patients be informed that their breasts will eventually sag following mastopexy. Procedures to prevent this from occurring include the use of permanent mesh encircling the breast mound. Mastopexy and reduction mammaplasty share similar operative strategies as well as complications. All techniques suffer bottoming out to different degrees.

Breast base diameter will change very little over time as long as the breast volume remains constant; eg, weight gain can increase breast volume.

An increase in the nipple-areola diameter is unlikely with vertical mastopexy; however, increased areola diameter is associated with periareolar mastopexy. To minimize this complication, a permanent purse-string suture is recommended. Suprasternal notch-to-inframammary crease distance changes very little in comparison with the nipple-to-inframammary crease distance.

The suprasternal notch-to-nipple distance changes very little postoperatively. When a prosthesis is used during mastopexy, this distance will increase; however, the nipple-to-inframammary crease will usually increase to a greater extent.

234
Q
A 45-year-old woman comes to the office for consultation regarding augmentation mammaplasty. She wears a size 32B brassiere; height is 5 ft 3 in (160 cm), and weight is 130 lb (59 kg). Subglandular placement of saline prostheses is planned. Which of the following is the primary advantage of using saline rather than silicone prostheses in this patient?
A)Easier detection of rupture
BLess capsular formation
C)Less wrinkling
D)Lighter prosthesis
E)Lower risk of leakage
A

A)Easier detection of rupture

Although both silicone and saline prostheses rupture at a similar rate, a saline rupture is more easily detectable because the saline is resorbed in the body. The deflated breast will be smaller in volume. Subtle changes, such as decreased upper pole fullness or increased softness, may be the only clues to silicone rupture on physical examination. Ultrasonography or MRI may be needed to confirm the diagnosis.

Saline prostheses are firmer than silicone; they are more likely to be palpable than silicone prostheses as well. Neither prosthesis has been associated with systemic immune syndromes, and both prostheses produce capsular contracture, wrinkling, and leakage.

235
Q

A 28-year-old woman comes for follow-up evaluation 2 weeks after undergoing bilateral augmentation mammaplasty with subpectoral placement of 325-mL, round, smooth saline prostheses. She is now concerned that both prostheses appear “too high.” Physical examination shows fullness in the upper quadrants of both breasts. Which of the following interventions is most appropriate?
A)Administration of oral zafirlukast
B)Application of a circumferential breast band
C)Injection of corticosteroid into the inframammary crease
D)Open capsulotomy
E)Percutaneous release of the inframammary crease

A

B)Application of a circumferential breast band

The most appropriate recommendation is breast band application. Breast shape following augmentation mammaplasty undergoes dynamic changes. The skin envelope and pectoralis muscle stretch under the expansion pressure of the prosthesis. The skin of the lower pole will stretch, allowing the prostheses to migrate inferiorly. Breast massage and a circumferential elastic breast band applied around the superior breast encourage this migration.

Zafirlukast is a leukotriene-antagonist that is used for the treatment of asthma. Preliminary studies suggest improvement in capsule contractures. This drug is associated with potential life-threatening liver complications as well as neuropsychiatric events. Because administration in the scenario described would constitute an off-label use of the drug, extensive discussion with the patient would be required prior to use.

In the past, steroid was injected into the saline compartment of a double-lumen prosthesis in an attempt to decrease the incidence of capsule contraction. This delivery system was uncontrolled and many prostheses migrated beyond the normal limits of the inframammary crease. Postoperative steroid injection has been used with some success for the prevention of recurrent capsule contracture following capsulectomy.

If residual inferior pectoralis muscle fibers are left intact along the rib or capsule contracture develops, open capsulotomy may be required; however, conservative treatment is indicated at this early postoperative period.

Percutaneous release would expose the patient to unnecessary complications of prosthesis injury, bleeding, and inframammary crease malposition.

236
Q
A 40-year-old woman comes to the office because of firmness of the right breast. Twenty years ago, she underwent augmentation mammaplasty with smooth silicone prostheses placed in subglandular pockets. Which of the following is the most appropriate management?
A)Injection of corticosteroids
B)Treatment with zafirlukast (Accolate)
C)Closed capsulotomy
D)Open capsulotomy
E)Total capsulectomy
A

E)Total capsulectomy

In the patient described with a capsular contracture, the most appropriate option is open capsulectomy. As opposed to open capsulotomy, open capsulectomy removes the entire capsule. Leaving the capsule behind in open capsulotomy can contribute to late seromas. Scar tissue left behind during an open capsulotomy may also prevent the prosthesis and breast from obtaining a natural shape.

Closed capsulotomy is no longer advised for breast prostheses because of the risk of rupturing the prosthesis during the procedure. Open capsulotomy and open capsulectomy with replacement of the prosthesis in the subglandular plane will continue to be associated with higher capsular contracture rates than submuscular or dual-plane placement. These are options for the patient as long as she understands the trade-offs of keeping the prosthesis in this plane.

Zafirlukast (Accolate) is a leukotriene receptor antagonist that is used as a bronchodilator in the management of asthma. The evidence supporting its use in capsular contracture is anecdotal. It is not approved by the US Food and Drug Administration (FDA) for use in capsular contracture; therefore, its use in the scenario described would be considered an ?off-label? indication. As such, zafirlukast cannot be recommended for the routine treatment of capsular contracture.

237
Q
A 24-year-old woman comes to the office 8 months after undergoing a circumareolar mastopexy/augmentation. She is concerned because her areolas are now asymmetric. They were symmetric preoperatively. Physical examination shows that the right areola diameter is 7 cm and the left areola diameter is 4 cm. The most likely cause of this asymmetry is a failure of which of the following?
A)Breast pillar approximation
B)Periareolar de-epithelialization
C)Prosthesis pocket
D)Purse-string suture
E)Skin envelope tailor tacking
A

D)Purse-string suture

The most likely cause of nipple-areola asymmetry in the patient described is failure in the purse-string suture. Periareolar mastopexy/augmentation has been plagued with inconsistent control of the nipple-areola complex diameter. This mastopexy technique creates concentric resection of periareolar epithelium to elevate the nipple-areola complex and reduce the skin envelope. The etiology of this areola-spreading is the tension of the closure intrinsic to the technique. Use of a permanent suture for the purse-string helps limit the postoperative spreading of the areolar diameter. Introduction of the interlocking polytetrafluoroethylene (GORE-TEX) suture has allowed improved control of areolar shape and diameter. If one of the purse-string sutures breaks or pulls through its dermal attachments, that areola will be subject to the forces of tension and expand in diameter. In the patient described, operative correction involves either replacing the purse-string on the widened side or removing the purse-string on the smaller diameter areola.

Periareolar de-epithelialization is the cause of the tension and is an essential part of the procedure. In patients who are significantly asymmetric, tension of the areolas will also be asymmetric; however, a permanent purse-string suture is crucial in these cases.

Prosthesis pocket and parenchyma shaping sutures will not have the impact on areolar diameter that is described in this scenario.

Envelope tailor tacking relates to final adjustments in periareolar de-epithelialization.

238
Q
A 32-year-old woman comes to the office for consultation regarding augmentation mammaplasty. She is concerned about the potential complications with the use of silicone gel prostheses within the first 5 years postoperatively. Which of the following is the most commonly reported complication of the implantation of cohesive silicone gel breast prostheses?
A)Capsular contracture
B)Granuloma
C)Hematoma
D)Infection
E)Rupture
A

A)Capsular contracture

Cohesive silicone gel is a breast prosthesis option that has been approved by the FDA since 2006. Cohesive gel prostheses have also been called ?gummy bear? prostheses. They maintain their shape because of the increased cross-linking within the silicone gel.

A study by Cunningham followed 1008 patients and 1898 cohesive gel prostheses. Rupture rate was 1.1% for aesthetics and 3.8% for reconstructive procedures. Capsular contracture rates (Baker III/IV) were 9.8/13.7%, and infection was 1.6/6.1%, respectively. Thus, capsular contracture was the most common of the listed complications. The reported incidence of hematoma is approximately 2%.

It should be noted that complications occur more commonly in primary reconstruction as compared to primary augmentation. These findings are important in the preoperative counseling of patients.